You are on page 1of 572

COMPLETE Cardio

Review for the USMLE


Step 2
by AJmonics
In this video...
200 cardiology review questions
for the USMLE Step 2 exam

We will be covering
EVERYTHING YOU NEED TO KNOW!!
You will be prepared on exam day!
Q1 Cardiology
A 37-year-old woman has left-sided chest pain that worsens with
movement of the left arm. Palpation of the chest reproduces the
pain. EKG is shown. What is the next best step?

A) Pacemaker
CC BY-SA 4.0, https://commons.wikimedia.org/w/index.php?curid=47645793

B) Observation
Q1 Cardiology
A 37-year-old woman has left-sided chest pain that worsens with
movement of the left arm. Palpation of the chest reproduces the
pain. EKG is shown. What is the next best step?

A) Pacemaker
CC BY-SA 4.0, https://commons.wikimedia.org/w/index.php?curid=47645793

B) Observation
Q1 Cardiology
A 37-year-old woman has left-sided chest pain that worsens with
movement of the left arm. Palpation of the chest reproduces the
pain. EKG is shown. What is the next best step?

A) Pacemaker
CC BY-SA 4.0, https://commons.wikimedia.org/w/index.php?curid=47645793

B) Observation - she likely has just muscle pain!


Q1 Cardiology
Ischemia produces a
-"sore", "squeezing", or "pressure-like" pain that does not
change with movement, palpation, or respiration!!!

Ischemia can effectively be ruled out if there is:


-pain with movement (positional)
-pain with palpation (tenderness)
-pain with respiration (pleuritic)
Q2 Cardiology
A 52-year-old woman recently started on a blood pressure
medication now complains of chest tightness, pain in her elbows
and knees, and erythematous plaques on her face, arms, and
torso. Which medication caused this to occur?

A) Lisinopril
B) Furosemide
C) Hydralazine
D) Verapamil
Q2 Cardiology
A 52-year-old woman recently started on a blood pressure
medication now complains of chest tightness, pain in her elbows
and knees, and erythematous plaques on her face, arms, and
torso. Which medication caused this to occur?

A) Lisinopril
B) Furosemide
C) Hydralazine
D) Verapamil
Q2 Cardiology
A 62-year-old woman recently started on a blood pressure
medication now complains of chest tightness, pain in her elbows
and knees, and erythematous plaques on her face, arms, and
torso. Which medication caused this to occur?

A) Lisinopril - dry cough, hyperkalemia, angioedema


B) Furosemide - hypotension, hypokalemia, ototoxicity
C) Hydralazine - flushing, headaches, drug-induced lupus
D) Verapamil - bradycardia, AV block, gingival hyperplasia
Q2 Cardiology
Causes of drug induced lupus:

MNEMONIC: *SHIPP*! Loops = Lupus

-Sulfasalazine
-Hydralazine
-Isoniazid
-Procainamide
-Penicillamine
Q3 Cardiology
A 61-year-old woman, with a history of chest pain, presents with
10 minutes of crushing substernal chest pain radiating to the left
arm. Troponin T levels = .4 ng/mL. What is the diagnosis?

A) Stable angina
B) Prinzmetal's angina
C) Unstable angina
D) Myocardial infarction
Q3 Cardiology
A 61-year-old woman, with a history of chest pain, presents with
10 minutes of crushing substernal chest pain radiating to the left
arm. Troponin T levels = .4 ng/mL. What is the diagnosis?

A) Stable angina
B) Prinzmetal's angina
C) Unstable angina
D) Myocardial infarction
Q3 Cardiology
A 61-year-old woman, with a history of chest pain, presents with
10 minutes of crushing substernal chest pain radiating to the left
arm. Troponin T levels = .4 ng/mL. What is the diagnosis?

A) Stable angina - chest pain that is reproducible with exertion


B) Prinzmetal's angina - vasospasm; no cardiac enzymes (tx: CCB)
C) Unstable angina - no cardiac enzymes (tx: similar to MI)
D) Myocardial infarction - because troponin .4ng/mL or above
Q4 Cardiology
A 50-year-old woman recently started taking
fluconazole and ondansetron. She now
develops palpitations; this is her EKG:

What is the proper treatment?


A) Adenosine
B) Magnesium
Q4 Cardiology
A 50-year-old woman recently started taking
fluconazole and ondansetron. She now
develops palpitations; this is her EKG:

What is the proper treatment?


A) Adenosine
B) Magnesium
Q4 Cardiology
A 50-year-old woman recently started taking
fluconazole and ondansetron. She now
develops palpitations; this is her EKG.

What is the proper treatment?


A) Adenosine
B) Magnesium *cardioversion indicated if patient is hemodynamically unstable*
Q4 Cardiology
Causes of Torsades de Pointes:
• antiArrhythmics (e.g., sotalol, quinidine)
• antiBiotics (e.g., flouroquinolones, macrolides)
• antiCychotics (e.g., risperidone)
• antiDepressants (e.g., SSRI's, TCA's)
• antiEmetics (e.g., ondansetron)
• antiFungals (e.g., fluconazole)
⚬ and various electrolyte abnormalities:
⚬ (​​↓magnesium, ↓potassium, ↓calcium)
Q4 Cardiology
Symptoms of Torsades de Pointes:
• typically palpitations, dizziness, and syncope
• can present with sudden cardiac death
Q5 Cardiology
A man comes to the ED with crushing chest pain for
the last hour; EKG shows ST elevations in V2-V4.
What is the next step?

A) CK-MB level
B) oxygen
C) thrombolytics
D) aspirin
Q5 Cardiology
A man comes to the ED with crushing chest pain for
the last hour; EKG shows ST elevations in V2-V4.
What is the next step?

A) CK-MB level
B) oxygen
C) thrombolytics
D) aspirin
Q5 Cardiology
A man comes to the ED with crushing chest pain for
the last hour; EKG shows ST elevations in V2-V4.
What is the next step?

A) CK-MB level - won't be elevated after an hour


B) oxygen - does not lower mortality
C) thrombolytics - only after 2 antiplatelets
D) aspirin - lowers mortality!
Q5 Cardiology
Time frame of diagnostic test
abnormalities
EKG: abnormal immediately
Myoglobin: 1 hour-2 days
CK-MB: 5 hours-2 days (*use for reinfarction*)
Troponin levels: 4 hours - 10 days
Q6 Cardiology
A 62-year-old woman with hypertension comes to the ED with
several days of chest pain and SOB. Now she is
unresponsive. EKG is shown. What is the next step?

A) Defibrillation
B) Cardioversion
C) Amiodarone
D) Magnesium sulfate
E) Epinephrine
Q6 Cardiology
A 62-year-old woman with hypertension comes to the ED with
several days of chest pain and SOB. Now she is
unresponsive. EKG is shown. What is the next step?

A) Defibrillation
B) Cardioversion
C) Amiodarone
D) Magnesium sulfate
E) Epinephrine
Q6 Cardiology
A 62-year-old woman with hypertension comes to the ED with
several days of chest pain and SOB. Now she is
unresponsive. EKG is shown. What is the next step?

A) Defibrillation - for VF or pulseless VT


B) Cardioversion - hemodynamically unstable tachyarr.
C) Amiodarone - hemodynamically stable patients w/ v-tach
D) Magnesium sulfate - in case of torsades de pointes
E) Epinephrine - every 3-5 minutes in patients with v-fib after defib & CPR
Q7 Cardiology
A 26-year-old athlete comes into the office for a routine
checkup. EKG shows sinus bradycardia, progressive
lengthening of the PR interval, followed by intermittent
dropped QRS complexes. No ST-segment abnormalities
are present.
What is the best next step?
A) Pacemaker
B) Reassurance
Q7 Cardiology
A 26-year-old athlete comes into the office for a routine
checkup. EKG shows sinus bradycardia, progressive
lengthening of the PR interval, followed by intermittent
dropped QRS complexes. No ST-segment abnormalities
are present.
What is the best next step?
A) Pacemaker
B) Reassurance
CC BY-SA 4.0, https://commons.wikimedia.org/w/index.php?curid=47645793
Q7 AV Block
Cardiology
First degree: asymptomatic [observation]
Second degree (MT1): usually
asymptomatic [usually observation]
Second degree (MT2): fatigue, light-
headedness, syncope [pacemaker]
Third degree: fatigue, light-headedness,
syncope [pacemaker]
CC BY-SA 4.0, https://commons.wikimedia.org/w/index.php?curid=47645793
Q8 Cardiology
A 46-year-old man has had frequent epigastric
burning after playing basketball for the past 6
months; antacids provide no relief. BP: 142/91,
no heart murmurs, abdomen is nontender.
EKG is normal. Next best step?
A) exercise EKG
B) H. pylori stool testing
Q8 Cardiology
A 46-year-old man has had frequent epigastric
burning after playing basketball for the past 6
months; antacids provide no relief. BP: 142/91,
no heart murmurs, abdomen is nontender.
EKG is normal. Next best step?
A) exercise EKG
B) H. pylori stool testing
Q8 Cardiology
A 46-year-old man has had frequent epigastric
burning after playing basketball for the past 6
months; antacids provide no relief. BP: 142/91,
no heart murmurs, abdomen is nontender.
EKG is normal. Next best step?
*Take home: Get exercise stress EKG in patients
with suspected stable ischemic heart disease!*
Q8 Cardiology
Angina Classification
Typical (or classic) angina consists of
(1) Substernal chest pain/discomfort that is
(2) Provoked by exertion or emotional stress and
(3) relieved by rest or nitroglycerine (or both).

Atypical angina applies when 2 out of 3 criteria of classic angina


are present (e.g., epigastric pain).
Q9 Cardiology
A 51-year-old woman with a history of hypertension comes to the
ED with chest pain and diaphoresis that has lasted for 24 hours.
EKG shows ST elevations in leads II, III, and AVF. Three days
later, she develops sudden SOB and low blood pressure, with
bibasilar crackles and a faint systolic murmur. Was is the cause of
the patient's new symptoms?

A) Papillary muscle rupture


B) LV free wall rupture
Q9 Cardiology
A 51-year-old woman with a history of hypertension comes to the
ED with chest pain and diaphoresis that has lasted for 24 hours.
EKG shows ST elevations in leads II, III, and AVF. Three days
later, she develops sudden SOB and low blood pressure, with
bibasilar crackles and a faint systolic murmur. Was is the cause of
the patient's new symptoms?

A) Papillary muscle rupture


B) LV free wall rupture
Q9 Cardiology
Q9 Cardiology

PAPPA likes RCA computers

FREE the LAD!


Q10 Cardiology
A 53-year-old woman is evaluated 5 days after an MI involving
the LAD. She suddently develops chest pain and EKG shows HR
of 140/min; she soon becomes unresponsive. What is the
diagnosis?

A) Free wall rupture


B) IV septum rupture
Q10 Cardiology
A 53-year-old woman is evaluated 5 days after an MI involving
the LAD. She suddently develops chest pain and EKG shows HR
of 140/min; she soon becomes unresponsive. What is the
diagnosis?

A) Free wall rupture


B) IV septum rupture
Q10 Cardiology
A 53-year-old woman is evaluated 5 days after an MI involving
the LAD. She suddently develops chest pain and EKG shows HR
of 140/min; she soon becomes unresponsive. What is the
diagnosis?

A) Free wall rupture


B) IV septum rupture - unlikely to cause
such a rapid progression to pulselessness
Q11 Cardiology
What is the pathophysiology of an S4 sound?

S4
A) ATP depletion
B) increased elasticity
Q11 Cardiology
What is the pathophysiology of an S4 sound?

S4
A) ATP depletion
B) increased elasticity
Q11 Cardiology
What is the pathophysiology of an S4 sound?

S4
A) ATP depletion
B) increased elasticity
Q11 Cardiology
Features of S4
-heard just before S1
-caused by blood hitting a stiff LV wall during atrial contraction
-can be normal in the elderly

Featues of S3
-heard just after S2
-caused by blood filling an enlarged LV cavity during passive distolic filling
-can be normal in children, adolescents, and during pregnancy
Q12 Cardiology
The most common cause of isolated aortic stenosis in
elderly patients is:

A) Genetics
B) Hypertension
C) Age related calcific changes
Q12 Cardiology
The most common cause of isolated aortic stenosis in
elderly patients is:

A) Genetics
B) Hypertension
C) Age related calcific changes
Q13 Cardiology
A 1-hour-old baby is evaluated for respiratory distress, with
95% oxygen on room air. The mother had poorly controlled
gestational diabetes. Echo shows a small left ventricular
cavity and an increased IV septum thickness.
What is the next best step?

A) Beta blocker therapy


B) Cardiac surgery
Q13 Cardiology
A 1-hour-old baby is evaluated for respiratory distress, with
95% oxygen on room air. The mother had poorly controlled
gestational diabetes. Echo shows a small left ventricular
cavity and an increased IV septum thickness.
What is the next best step?

A) Beta blocker therapy


B) Cardiac surgery
Q13 Cardiology
A 1-hour-old baby is evaluated for respiratory distress, with
95% oxygen on room air. The mother had poorly controlled
gestational diabetes. Echo shows a small left ventricular
cavity and an increased IV septum thickness.
What is the next best step?

A) Beta blocker therapy - for HCM


B) Cardiac surgery - for fixed LVOT obstruction
Q14 Cardiology
A 54-year-old woman with a recent URI comes in with worsening
exertional dyspnea for several days. BP: 86/59, Pulse: 109, JVD:
11 cm H20; lungs are clear and heart sounds are muffled.
What is the cause of the patient's symptoms?

A) Reduced cardiac contractility


B) Decreased LV preload
Q14 Cardiology
A 54-year-old woman with a recent URI comes in with worsening
exertional dyspnea for several days. BP: 86/59, Pulse: 109, JVD:
11 cm H20; lungs are clear and heart sounds are muffled.
What is the cause of the patient's symptoms?

A) Reduced cardiac contractility


B) Decreased LV preload
Q14 Cardiology
A 54-year-old woman with a recent URI comes in with worsening
exertional dyspnea for several days. BP: 86/59, Pulse: 109, JVD:
11 cm H20; lungs are clear and heart sounds are muffled.
What is the cause of the patient's symptoms?

A) Reduced cardiac contractility


B) Decreased LV preload - restricted venous return
Q15 Cardiology
A 28-year-old woman at 25 weeks gestation comes in with
a blood pressure of 153/89 mmHg (154/90 mmHg on
repeat). Physical exam of mom is normal, and baby's
heart rate is 150/min on Doppler. Liver function studies
are normal. What is the next step?

A) Magnesium sulfate
B) Reassurance
C) 24-urine collection (for total protein)
Q15 Cardiology
A 28-year-old woman at 25 weeks gestation comes in with
a blood pressure of 153/89 mmHg (154/90 mmHg on
repeat). Physical exam of mom is normal, and baby's
heart rate is 150/min on Doppler. Liver function studies
are normal. What is the next step?

A) Magnesium sulfate
B) Reassurance
C) 24-urine collection (for total protein)
Q15 Cardiology
A 28-year-old woman at 25 weeks gestation comes in with
a blood pressure of 153/89 mmHg (154/90 mmHg on
repeat). Physical exam of mom is normal, and baby's
heart rate is 150/min on Doppler. Liver function studies
are normal. What is the next step?

A) Magnesium sulfate - preeclampsia (severe)


B) Reassurance - no, need to evaluate
C) 24-urine collection (for total protein)
Q16 Cardiology
Correcting which of the following CAD risk factors
will benefit the patient the most?

A) DM
B) smoking
C) hypertension
D) hyperlipidemia
E) weight loss
Q16 Cardiology
Correcting which of the following CAD risk factors
will benefit the patient the most?

A) DM
B) smoking
C) hypertension
D) hyperlipidemia
E) weight loss
Q16 Cardiology
Correcting which of the following CAD risk factors
will benefit the patient the most?

A) DM
B) smoking - 1 year: 50%; 2 years: 90%!!!
C) hypertension
D) hyperlipidemia
E) weight loss
Q17 Cardiology
According to current guidelines, who does NOT require
statin therapy?

A) A patient with a history of stroke


B) A patient with a history of stable angina
C) A patient with an LDL greater than 190 mg/dL
D) A diabetic patient older than 40
E) A patient with an ASCVD 10-year risk of 5%
Q17 Cardiology
According to current guidelines, who does NOT require
statin therapy?

A) A patient with a history of stroke


B) A patient with a history of stable angina
C) A patient with an LDL greater than 190 mg/dL
D) A diabetic patient older than 40
E) A patient with an ASCVD 10-year risk of 5%
Q17 Cardiology
According to current guidelines, who does NOT require
statin therapy?

A) A patient with a history of stroke


B) A patient with a history of stable angina
C) A patient with an LDL greater than 190 mg/dL
D) A diabetic patient older than 40
E) A patient with an ASCVD 10-year risk of 5% - must be at
least 7.5%-10%
Q18 Cardiology
What is the most common cause of stent thrombosis?

A) Medication noncompliance
B) Incomplete stent apposition
Q18 Cardiology
What is the most common cause of stent thrombosis?

A) Medication noncompliance
B) Incomplete stent apposition
Q18 Cardiology
What is the most common cause of stent thrombosis?

A) Medication noncompliance
B) Incomplete stent apposition

*Make sure this doesn't happen!! Stent


thrombosis has a very high mortality
rate!! (about 50%)*
Q19 Cardiology
A 45-year-old woman from Africa presents with exertional
dyspnea, an early diastolic sound (i.e., an opening "snap")
followed by a rumbling diastolic murmur. Which of the following
is expected to be seen upon measuring the LV diastolic
pressure?

A) Increased pressure
B) Normal pressure
Q19 Cardiology
A 45-year-old woman from Africa presents with exertional
dyspnea, an early diastolic sound (i.e., an opening "snap")
followed by a rumbling diastolic murmur. Which of the following
is expected to be seen upon measuring the LV diastolic
pressure?

A) Increased pressure
B) Normal pressure
Q20 Cardiology
A 74-year-old woman presents to her family doctor with lower
face "puffiness", but no other complaints. Her medications
include aspirin, sacubitril-valsartan, furosemide, and a statin.
Creatinine and electrolyte levels are normal. What is the next
step?

A) Increase furosemide
B) Go to the emergency room
Q20 Cardiology
A 74-year-old woman presents to her family doctor with lower
face "puffiness", but no other complaints. Her medications
include aspirin, sacubitril-valsartan, furosemide, and a statin.
Creatinine and electrolyte levels are normal. What is the next
step?

A) Increase furosemide
B) Go to the emergency room
Q20 Cardiology
A 74-year-old woman presents to her family doctor with lower
face "puffiness", but no other complaints. Her medications
include aspirin, sacubitril-valsartan, furosemide, and a statin.
Creatinine and electrolyte levels are normal. What is the next
step?

A) Increase furosemide
B) Go to the emergency room - angioedema
is an adverse effect of sacubitril-valsartan
Q21 Cardiology
A 56-year-old man with pleuritic chest pain presents 3 days after
an MI. Lungs are clear bilaterally. EKG shows tachycardia and
diffuse PR segment depression. What is the next best step?

A) Echocardiogram
B) Exercise stress test
C) Troponin levels
Q21 Cardiology
A 56-year-old man with pleuritic chest pain presents 3 days after
an MI. Lungs are clear bilaterally. EKG shows tachycardia and
diffuse PR segment depression. What is the next best step?

A) Echocardiogram
B) Exercise stress test
C) Troponin levels
Q21 Cardiology
A 56-year-old man with pleuritic chest pain presents 3 days after
an MI. Lungs are clear bilaterally. EKG shows tachycardia and
diffuse PR segment depression. What is the next best step?

A) Echocardiogram - to assess pericardial effusion


B) Exercise stress test
C) Troponin levels
Q22 Cardiology
A 3-day-old girl is evaluated for poor feeding and fusiness. Blood
pressure is 65/40 mmHg in the right upper arm and 39/27 in the
lower right leg. What is the next best step?

A) Initiate epinephrine
B) Administer prostaglandin E1
Q22 Cardiology
A 3-day-old girl is evaluated for poor feeding and fusiness. Blood
pressure is 65/40 mmHg in the right upper arm and 39/27 in the
lower right leg. What is the next best step?

A) Initiate epinephrine
B) Administer prostaglandin E1
Q22 Cardiology
A 3-day-old girl is evaluated for poor feeding and fusiness. Blood
pressure is 65/40 mmHg in the right upper arm and 39/27 in the
lower right leg. What is the next best step?

A) Initiate epinephrine
B) Administer prostaglandin E1 - maintains
patency of the ductus arteriosus
Q23 Cardiology
A 10-hour-old boy is evaluated in the nursery. Pulse ox= 99%.
The baby's trunk is pink but hands and feet are cyanotic. There is
a low pitched 2/6 midsystolic ejection murmur heard at the upper
left sternal border. Pulses are symmetric and normal. What is the
next best step?

A) Reassurance
B) Echocardiogram
Q23 Cardiology
A 10-hour-old boy is evaluated in the nursery. Pulse ox= 99%.
The baby's trunk is pink but hands and feet are cyanotic. There is
a low pitched 2/6 midsystolic ejection murmur heard at the upper
left sternal border. Pulses are symmetric and normal. What is the
next best step?

A) Reassurance
B) Echocardiogram
Q23 Cardiology
A 10-hour-old boy is evaluated in the nursery. Pulse ox= 99%.
The baby's trunk is pink but hands and feet are cyanotic. There is
a low pitched 2/6 midsystolic ejection murmur heard at the upper
left sternal border. Pulses are symmetric and normal. What is the
next best step?

A) Reassurance
B) Echocardiogram
*Features of benign murmurs in newborn include soft
intensity, low-pitched (or musical), early/midsystolic*
Q24 Cardiology
A 76-year-old man with no complaints of cardiac or respiratory
symptoms is evaluated for a heart murmur. Echo shows severe
aortic stenosis with an the EF is 45% with a valve area of 1 cubic
cm. What is the best next step?

A) Recommend surgery
B) Reassurance and follow up
Q24 Cardiology
A 76-year-old man with no complaints of cardiac or respiratory
symptoms is evaluated for a heart murmur. Echo shows severe
aortic stenosis with an the EF is 45% with a valve area of 1 cubic
cm. What is the best next step?

A) Recommend surgery
B) Reassurance and follow up
Q24 Cardiology
Indications for Aortic Valve Replacement

Severe aortic stenosis & ≥1 of the following:


• Symptoms (eg, angina, syncope)
• LV ejection fraction <50%
• Undergoing other cardiac surgery (e.g., CABG)
Q25 Cardiology
A 51-year-old woman without cardiac symptoms is evaluated for
a heart murmurs. Echo shows MR with an EF = 51%. What is the
next best step?

A) Reassurance
B) Beta blocker
C) Surgery
Q25 Cardiology
A 51-year-old woman without cardiac symptoms is evaluated for
a heart murmurs. Echo shows MR with an EF = 51%. What is the
next best step?

A) Reassurance
B) Beta blocker
C) Surgery
Q25 Cardiology
A 51-year-old woman without cardiac symptoms is evaluated for
a heart murmurs. Echo shows MR with an EF = 51%. What is the
next best step?

A) Reassurance
B) Beta blocker
C) Surgery - *MR with an EF 30%-60% is
REALLY bad! The regurgitant flow accounts
for a large amount of the stroke volume!*
Q26 Cardiology
A 1-month-old boy is brought it for a well visit. The baby is
breathing well and is not cyanotic. A 3/6 holosystolic murmur and
thrill are present at the left lower sternal border. CXR shows an
enlarged left side of the heart. What is the pathophysiology of this
finding?

A) Left atrial preload increase


B) Asymmetric septal hypertrophy
Q26 Cardiology
A 1-month-old boy is brought it for a well visit. The baby is
breathing well and is not cyanotic. A 3/6 holosystolic murmur and
thrill are present at the left lower sternal border. CXR shows an
enlarged left side of the heart. What is the pathophysiology of this
finding?

A) Left atrial preload increase


B) Asymmetric septal hypertrophy
Q27 Cardiology
The most common cause of constrictive pericarditis in developing
countries is ___.

A) Pneumoconiosis
B) Tuberculosis
Q27 Cardiology
The most common cause of constrictive pericarditis in developing
countries is ___.

A) Pneumoconiosis
B) Tuberculosis
Q27 Cardiology
The most common cause of constrictive pericarditis in developing
countries is ___.

A) Pneumoconiosis
B) Tuberculosis
*Constrictive pericarditis impairs ventricular filling;
symptoms related to decreased cardiac output (e.g.,
fatigue and dyspnea on exertion) and signs of
volume overload*
Q28 Cardiology
Which imaging is used to confirm a diagnosis of aortic dissection
in a distressed hypotensive patient?

A) MR angiography
B) Transesophageal echo
Q28 Cardiology
Which imaging is used to confirm a diagnosis of aortic dissection
in a distressed hypotensive patient?

A) MR angiography
B) Transesophageal echo
Q28 Cardiology
Which imaging is used to confirm a diagnosis of aortic dissection
in a distressed hypotensive patient?

A) MR angiography
B) Transesophageal echo
Ascending aortic dissection necessitates emergency
surgical repair; imaging with TEE (or CT
angiography in non-hypotensive patients) first
confirms the diagnosis and helps guide surgery.
Q29 Cardiology
What is the most common cause of acute cardiac arrest
subsequent to a myocardial infarction?

A) Ventricular fibrillation
B) Pulseless electrical activity
Q29 Cardiology
What is the most common cause of acute cardiac arrest
subsequent to a myocardial infarction?

A) Ventricular fibrillation
B) Pulseless electrical activity
Q30 Cardiology
Which of the following women CAN get pregnant?

A) A woman with symptomatic mitral stenosis


B) A woman with symptomatic aortic stenosis
C) A woman with heart failure with LVEF = 50%
D) a woman with pulmonary arterial hypertension
Q30 Cardiology
Which of the following women CAN get pregnant?

A) A woman with symptomatic mitral stenosis


B) A woman with symptomatic aortic stenosis
C) A woman with heart failure with LVEF = 50%
D) a woman with pulmonary arterial hypertension
Q30 Cardiology
Which of the following women CAN get pregnant?

A) A woman with symptomatic mitral stenosis


B) A woman with symptomatic aortic stenosis
C) A woman with heart failure with LVEF = 50% (less than 30%)
D) a woman with pulmonary arterial hypertension
Q30 Cardiology
Cardiovascular Contraindications to Pregnancy

MAP =
-mitral stenosis (symptomatic),
-aortic stenosis (symptomatic),
-pulmonary arterial hypertension
Q30 Cardiology
Cardiovascular Contraindications to Pregnancy

MAP =
-mitral stenosis (symptomatic),
-aortic stenosis (symptomatic),
-pulmonary arterial hypertension
Q31 Cardiology
A new diastolic murmur is heard along the left sternal border in a
patient several years after aortic valve replacement. What is the
next best step?

A) Reassurance & follow up


B) Echocardiogram
Q31 Cardiology
A new diastolic murmur is heard along the left sternal border in a
patient several years after aortic valve replacement. What is the
next best step?

A) Reassurance & follow up


B) Echocardiogram
Q31 Cardiology
A new diastolic murmur is heard along the left sternal border in a
patient several years after aortic valve replacement. What is the
next best step?

A) Reassurance & follow up


B) Echocardiogram - need to assess
for aortic regurgitation
Q32 Cardiology
A patient who hasn't seen a physician in years
presents to the ED with pressure-like chest pain.
BP = 175/99. EKG looks normal. 25 minutes later
the patient is evaluated and reports the same chest
pain. What is the next best step?

A) Cath lab
B) Repeat EKG
Q32 Cardiology
A patient who hasn't seen a physician in years
presents to the ED with pressure-like chest pain.
BP = 175/99. EKG looks normal. 25 minutes later
the patient is evaluated and reports the same chest
pain. What is the next best step?

A) Cath lab
B) Repeat EKG
Q32 Cardiology
A patient who hasn't seen a physician in years
presents to the ED with pressure-like chest pain.
BP = 175/99. EKG looks normal. 25 minutes later
the patient is evaluated and reports the same chest
pain. What is the next best step?

A) Cath lab
B) Repeat EKG - should be repeated every
15-20 minutes in unstable patient
Q33 Cardiology
A healthy-looking asymptomatic newborn is
being evaluated. BP = 81/53. A 2/6
holosystolic murmur is heard at the left lower
sternal border. What is the best next step?

A) Reassurance and follow up


B) Echocardiogram
Q33 Cardiology
A healthy-looking asymptomatic newborn is
being evaluated. BP = 81/53. A 2/6
holosystolic murmur is heard at the left lower
sternal border. What is the best next step?

A) Reassurance and follow up


B) Echocardiogram
Q34 Cardiology
The most significant controllable risk factor for
stroke is:

A) Blood glucose reduction


B) Blood pressure reduction
C) Smoking cessation
D) Weight loss
E) Serum cholesterol reduction
Q34 Cardiology
The most significant controllable risk factor for
stroke is:

A) Blood glucose reduction


B) Blood pressure reduction
C) Smoking cessation
D) Weight loss
E) Serum cholesterol reduction
Q35 Cardiology
A patient with a syphilis chancre, if left
untreated, can most likely develop:

A) Mitral valve stenosis


B) Ascending aortic aneurysm
C) Endocarditis
D) Free wall rupture
Q35 Cardiology
A patient with a syphilis chancre, if left
untreated, can most likely develop:

A) Mitral valve stenosis


B) Ascending aortic aneurysm
C) Endocarditis
D) Free wall rupture
Q35 Cardiology
A patient with a syphilis chancre, if left untreated, can
most likely develop:

A) Mitral valve stenosis - seen in late rheumatic heart


disease
B) Ascending aortic aneurysm
C) Endocarditis - not due to syphilis
D) Free wall rupture - seen 4-10 days after an MI
Q36 Cardiology
A 5-year-old boy who fainted is evaluated; EKG is shown. What
medication should be given?

CC BY-SA 3.0, https://commons.wikimedia.org/w/index.php?curid=24370863

A) Sotalol
B) Quinidine
C) Propranolol
Q36 Cardiology
A 5-year-old boy who fainted is evaluated; EKG is shown. What
medication should be given?

CC BY-SA 3.0, https://commons.wikimedia.org/w/index.php?curid=24370863

A) Sotalol
B) Quinidine
C) Propranolol
Q36 Cardiology
A 5-year-old boy who fainted is evaluated; EKG is shown. What
medication should be given?

Congenital long QT syndrome

CC BY-SA 3.0, https://commons.wikimedia.org/w/index.php?curid=24370863

A) Sotalol - contraindicated in LQTS


B) Quinidine - contraindicated in LQTS
C) Propranolol - also avoid exacerbating factors
Q37 Cardiology
A patient with angina is provided a daily medication to help
prevent episodes of angina. What is the mechanism of the
medication?

A) Decreased cardiac preload


B) Decreased myocardial contractility
Q37 Cardiology
A patient with angina is provided a daily medication to help
prevent episodes of angina. What is the mechanism of the
medication?
3 main medication classes for the
prevention of stable angina symptoms:
• beta blockers
• calcium channel blockers
• long-acting nitrates.
A) Decreased cardiac preload
B) Decreased myocardial contractility
Q37 Cardiology
Chronic Stable
Angina Treatment
Beta blockers
• First-line therapy Nitrates
• ↓ Myocardial contractility & HR • Long-acting; added for persistent angina
Nondihydropyridine CCBs • ↓ Preload through dilation of veins
• Alternative to beta blocker Ranolazine
• ↓ Myocardial contractility & HR • Alternative for refractory angina
Dihydropyridine CCBs • ↓ Myocardial calcium influx
• Added to beta blocker when in need
• Coronary artery vasodilation
• ↓ Afterload - systemic vasodilation
Q38 Cardiology
A systolic murmur at the apex of the heart that
disappears when squatting is:

A) mitral valve prolapse


B) ventricular septal defect
Q38 Cardiology
A systolic murmur at the apex of the heart that
disappears when squatting is:

A) mitral valve prolapse


B) ventricular septal defect
Q39 Cardiology
A 36-year-old female presents with a
holosystolic murmur that is loudest at the
apex. She complains of palpitations. What is
the cause of her condition?

A) Degeneration of the mitral valve


B) Rheumatic heart disease
Q39 Cardiology
A 36-year-old female presents with a
holosystolic murmur that is loudest at the
apex. She complains of palpitations. What is
the cause of her condition?

A) Degeneration of the mitral valve


B) Rheumatic heart disease
Q39 Cardiology
A 36-year-old female presents with a
holosystolic murmur that is loudest at the
apex. She complains of palpitations. What is
the cause of her condition?

A) Degeneration of the mitral valve


B) Rheumatic heart disease
Mitral valve prolapse is the most common cause of
chronic MR in developed countries; palpitations
seen in the chronic form
Q40 Cardiology
A 24-year-old woman presents to the ED due to intermittent
dizziness and weakness for several days. She has no
significant medical history and physical exam is otherwise
normal. EKG is shown below. Echocardiogram shows no
structural abnormalities. Which test is appropriate to be
ordered?

A) CT angiogram
B) Serum TSH level
https://commons.wikimedia.org/wiki/File:Afib_ecg_%28CardioNetworks_ECGpedia%29.jpg
Q40 Cardiology
A 24-year-old woman presents to the ED due to intermittent
dizziness and weakness for several days. She has no
significant medical history and physical exam is otherwise
normal. EKG is shown below. Echocardiogram shows no
structural abnormalities. Which test is appropriate to be
ordered?

A) CT angiogram
B) Serum TSH level
https://commons.wikimedia.org/wiki/File:Afib_ecg_%28CardioNetworks_ECGpedia%29.jpg
Q40 Cardiology
A 24-year-old woman presents to the ED due to intermittent
dizziness and weakness for several days. She has no
significant medical history and physical exam is otherwise
normal. EKG is shown below. Echocardiogram shows no
structural abnormalities. Which test is appropriate to be
ordered?

A) CT angiogram
B) Serum TSH level - hyperthyroidism can increase
atrial conduction automaticity - leads to AF
Q41 Cardiology
Symptomatic bradycardia is treated with:

A) Adenosine
B) Amiodarone
C) Atropine
D) Isoproterenol
E) Metoprolol
Q41 Cardiology
Symptomatic bradycardia is treated with:

A) Adenosine
B) Amiodarone
C) Atropine
D) Isoproterenol
E) Metoprolol
Q41 Cardiology
Symptomatic bradycardia is treated with:

A) Adenosine - AV nodal reentrant tachy.


B) Amiodarone - arrythmias
C) Atropine - symptomatic bradycardia
D) Isoproterenol - also for brady, but not 1st
E) Metoprolol - slows the heart further!
Q42 Cardiology
A 36-year-old male comes to the office and it is discovered on
EKG that he has premature atrial complexes. He drinks 2 cups
of coffee per day. He has no chest pain or other cardiac
symptoms. His father died of an MI at age 65. What is the next
best step?

A) 24-hour Holter monitoring


B) Patient should smoke marijuana
C) Patient should avoid caffeine
Q42 Cardiology
A 36-year-old male comes to the office and it is discovered on
EKG that he has premature atrial complexes. He drinks 2 cups
of coffee per day. He has no chest pain or other cardiac
symptoms. His father died of an MI at age 65. What is the next
best step?

A) 24-hour Holter monitoring


B) Patient should smoke marijuana
C) Patient should avoid caffeine
Q42 Cardiology
Advise to patients with PAC's:
"Please Avoid Caffeine & Smoking"
Q43 Cardiology
High dose niacin therapy (e.g., to treat lipid
abnormalities) often produces:

A) cutaneous flushing and itching


B) muscle pain and weakness
Q43 Cardiology
High dose niacin therapy (e.g., to treat lipid
abnormalities) often produces:

A) cutaneous flushing and itching


B) muscle pain and weakness
Q43 Cardiology
High dose niacin therapy (e.g., to treat lipid
abnormalities) often produces:

A) cutaneous flushing and itching


B) muscle pain and weakness
*Note: niacin is the most effective lipid modifying drug for
raising HDL levels; it reduces triglycerides and VLDL and
LDL synthesis primarily by reducing fatty acid
mobilization in fatty acid tissue; flushing and itching
occur due to vasodilation*
Q44 Cardiology
Aortic stenosis with a valve area of 2.5 cm2 generally
leads to exertional chest discomfort and shortness of
breath. T/F.

A) True
B) False
Q44 Cardiology
Aortic stenosis with a valve area of 2.5 cm2 generally
leads to exertional chest discomfort and shortness of
breath. T/F.

A) True
B) False
Q44 Cardiology
Aortic stenosis with a valve area of 2.5 cm2 generally
leads to exertional chest discomfort and shortness of
breath. T/F.

A) True
B) False - symptoms don't appear until
the valve area is less than 1 cubic cm.
(Normal area = 3-4 cubic cm.)
Q45 Cardiology
A woman develops progressive dyspnea, nonproductive
cough, and bilateral inspiratory crackles. Which
medication caused this?

A) Amiodarone
B) Beta blocker
C) Cyclophosphamide
D) Digoxin
Q45 Cardiology
A woman develops progressive dyspnea, nonproductive
cough, and bilateral inspiratory crackles. Which
medication caused this?

A) Amiodarone
B) Beta blocker
C) Cyclophosphamide
D) Digoxin
Q45 Cardiology
A woman develops progressive dyspnea, nonproductive
cough, and bilateral inspiratory crackles. Which
medication caused this?

A) Amiodarone- AV block, hypotension, pulmonary fib.


B) Beta blocker- asthma exacerbation, hypotension
C) Cyclophosphamide- hem. cystitis, bladder cancer
D) Digoxin- yellow vision, arrythmias, GI disturbances
Q46 Cardiology
Which of the following medications elevates blood
pressure (in 5% of patients)?

A) Oral contraceptives
B) Prazosin
Q46 Cardiology
Which of the following medications elevates blood
pressure (in 5% of patients)?

A) Oral contraceptives
B) Prazosin
Q46 Cardiology
Which of the following medications elevates blood
pressure in 5% of patients?

A) Oral contraceptives
B) Prazosin - used to treat
hypertension, BPH, PTSD night-
mares, and Raynauds
Q47 Cardiology
An infant with hemodynamic instability who underwent
cardiac surgery now develops tachycardia, tachypnea,
distant heart sounds, and enlargement of the cardiac
silhouette on CXR. What is the diagnosis?

A) Pericardial effusion
B) Mediastinitis
Q47 Cardiology
An infant with hemodynamic instability who underwent
cardiac surgery now develops tachycardia, tachypnea,
distant heart sounds, and enlargement of the cardiac
silhouette on CXR. What is the diagnosis?

A) Pericardial effusion
B) Mediastinitis
Q47 Cardiology
An infant with hemodynamic instability who underwent
cardiac surgery now develops tachycardia, tachypnea,
distant heart sounds, and enlargement of the cardiac
silhouette on CXR. What is the diagnosis?
Beck's triad (hypotension, distended
neck veins and muffled heart sounds)
A) Pericardial effusion
B) Mediastinitis
Q48 Cardiology
A few days after a viral infection a 45-year-old male
develops a low-grade fever, substernal chest pain that
worsens with deep breathing, and a scratchy triphasic
sound on auscultation. What is the first-line treatment?
A) Heparin
B) Naproxen + prednisone
C) Nitroglycerin
D) Ibuprofen + colchicine
Q48 Cardiology
A few days after a viral infection a 45-year-old male
develops a low-grade fever, substernal chest pain that
worsens with deep breathing, and a scratchy triphasic
sound on auscultation. What is the first-line treatment?
A) Heparin
B) Naproxen + prednisone
C) Nitroglycerin
D) Ibuprofen + colchicine
Q48 Cardiology
A few days after a viral infection a 45-year-old male
develops a low-grade fever, substernal chest pain that
worsens with deep breathing, and a scratchy triphasic
sound on auscultation. What is the first-line treatment?
A) Heparin - for acute coronary syndrome
B) Naproxen + prednisone - better for arthritis?
C) Nitroglycerin - for pain in acute coronary syndrome
D) Ibuprofen + colchicine - for viral pericarditis
Q49 Cardiology
A hemodynamically stable patient with BP = 170/100 has
an EKG shown below. What is the next step?

A) Carotid massage
B) Cardioversion
C) Diltiazem
Q49 Cardiology
A hemodynamically stable patient with BP = 170/100 has
an EKG shown below. What is the next step?

A) Carotid massage
B) Cardioversion
C) Diltiazem
Q49 Cardiology
A hemodynamically stable patient with BP = 170/100 has
an EKG shown below. What is the next step?

A) Carotid massage
B) Cardioversion
C) Diltiazem - OR a beta blocker (metoprolol); goal rate
should be less than 110/min
Q50 Cardiology
A newborn presents with micrognathia, microcephaly, and
rocker bottom feet. What cardiac defect does he likely
have?

A) Transposition of the Great Arteries


B) VSD
Q50 Cardiology
A newborn presents with micrognathia, microcephaly, and
rocker bottom feet. What cardiac defect does he likely
have?

A) Transposition of the Great Arteries


B) VSD
Q51 Cardiology
Pregnant lady + dyspnea + orthopnea + new AF=

A) Rheumatic mitral stenosis


B) Constrictive pericarditis
Q51 Cardiology
Pregnant lady + dyspnea + orthopnea + new AF=

A) Rheumatic mitral stenosis


B) Constrictive pericarditis
Q51 Cardiology
Pregnant lady + dyspnea + orthopnea + new AF=

A) Rheumatic mitral stenosis


B) Constrictive pericarditis
*Most likely pulmonary edema with decompensation due to
development of new afib (which worses diastolic filling
time, further increasing LA pressure, thus worsening the
pulmonary edema)*
Q52 Cardiology
Once malignant pericardial effusions are managed (e.g.,
pericardiocentesis), how are they prevented in the future
from recurrence?

A) Pericardial window
B) Prednisone
Q52 Cardiology
Once malignant pericardial effusions are managed (e.g.,
pericardiocentesis), how are they prevented in the future
from recurrence?

A) Pericardial window
B) Prednisone
Q52 Cardiology
Once malignant pericardial effusions are managed (e.g.,
pericardiocentesis), how are they prevented in the future
from recurrence?

A) Pericardial window - surgical removal of part of the pericardium to allow


fluid to drain into the pleural or peritoneal cavity (alternative: prolonged
pericardial catheter drain)
B) Prednisone - not initial therapy; require prolonged drainage
Q53 Cardiology
What is statin's mechanism of
action?

A) Enzyme blockage extracellularly


B) Inhibition of intracellular synthesis
pathway
Q53 Cardiology
What is statin's mechanism of
action?

A) Enzyme blockage extracellularly


B) Inhibition of intracellular synthesis
pathway
Q53 Cardiology
What is statin's mechanism of
action?

A) Enzyme blockage extracellularly


B) Inhibition of intracellular synthesis
pathway
*Statins inhibit HMG-CoA reductase,
required for cholestrol synthesis*
Q54 Cardiology
Sinus sick syndrome is most commonly
caused by:

A) Degeneration of the sinus node


B) Coronary artery occlusion
Q54 Cardiology
Sinus sick syndrome is most commonly
caused by:

A) Degeneration of the sinus node


B) Coronary artery occlusion
Q54 Cardiology
Sinus sick syndrome is most commonly caused
by:

A) Degeneration of the sinus node


-aka "sinus node dysfunction"; i.e., the SA node
can't generate an adequate heart rate
B) Coronary artery occlusion
Q55 Cardiology
An otherwise healthy looking newborn has cyanosis
present on his lips, trunk, and extremities. Oxygen
saturation is 75% on room air as well as after pure oxygen
is delivered. No stridor or murmurs are heard. What is the
diagnosis?

A) Vascular rings
B) Transposition of the great arteries
Q55 Cardiology
An otherwise healthy looking newborn has cyanosis
present on his lips, trunk, and extremities. Oxygen
saturation is 75% on room air as well as after pure oxygen
is delivered. No stridor or murmurs are heard. What is the
diagnosis?

A) Vascular rings
B) Transposition of the great arteries
Q55 Cardiology
Transposition of the Great Arteries
-central/peripheral cyanosis that does not improve with
pure oxygen because the problem is the heart structure,
not the lungs (O2 would not improve vascular ring, but
infant would present with stridor)
-in TotGA, the aorta comes from the RV and the PA from
the LV
A) Vascular rings
B) Transposition of the great arteries
Q56 Cardiology
A 56-year-old presents with pain, pallor, paresthesia,
pulselessness, and poikolothermia in right leg. What is
the next step?

A) IV heparin
B) Ankle brachial index
Q56 Cardiology
A 56-year-old presents with pain, pallor, paresthesia,
pulselessness, and poikolothermia in right leg.

A) IV heparin
B) Ankle brachial index
Q56 Cardiology
A 56-year-old presents with pain, pallor, paresthesia,
pulselessness, and poikolothermia in right leg.
classic presentation of a acute limb ischemia!!

IV heparin - patients with ALI diagnosed via clinical exam should be


immediately given anticoagulation (e.g., IV heparin);
for some patients this improves symptoms; for others, percutaneous
thrombolysis [alteplase] or thrombectomy is required)
Q57 Cardiology
What post-MI complication typically occurs 3-5 days after
the MI and presents with sudden onset cardiogenic shock
with hypotension and a new harsh holosystolic murmur
with a palpable thrill at the left sternal border?

A) Rupture of the IV septum


B) Papillary muscle rupture
Q57 Cardiology
What post-MI complication typically occurs 3-5 days after
the MI and presents with sudden onset cardiogenic shock
with hypotension and a new harsh holosystolic murmur
with a palpable thrill at the left sternal border?

A) Rupture of the IV septum


B) Papillary muscle rupture
Q57 Cardiology

A) Rupture of the IV septum - typically 3-5 days after MI;


presents with sudden-onset cardiogenic shock with
hypotension with a new harsh VSD-like murmur
B) Papillary muscle rupture - leads to acute severe MR,
along with hypotension, but the systolic murmur of the
MR is soft with no palpable thrill
Q58 Cardiology
Patients with WPW who develop a-fib should be treated
with:

A) Calcium channel blockers


B) Digoxin
C) Procainamide
Q58 Cardiology
Patients with WPW who develop a-fib should be treated
with:

A) Calcium channel blockers


B) Digoxin
C) Procainamide
Q58 Cardiology
Patients with WPW who develop a-fib should be treated
with:

A) Calcium channel blockers


B) Digoxin
C) Procainamide - electrical cardioversion if patient is
hemodynamically unstable
Q58 Cardiology
Patients with WPW who develop a-fib should be treated
with:

A) Calcium channel blockers


B) Digoxin
C) Procainamide - electrical cardioversion if patient is
hemodynamically unstable
*Atrial fibrillation (AF) occurs in 20% of individuals with WPW, and is a potentially life-
threatening emergency*
Q59 Cardiology
A patient with PAD will most like develop _ over the next
several years.

A) MI
B) Amputation
Q59 Cardiology
A patient with PAD will most like develop _ over the next
several years.

A) MI
B) Amputation
Q59 Cardiology
A patient with PAD will most like develop _ over the next
several years.

*PAD*
leg coldness, intermittent claudication, diminished pulses,
abnormal (<1) ABI; a strong predictor (20%) of future risk
of cardiovascular mortality; 1-2% progress to gangrene
requiring amputation
Q60 Cardiology
A patient who presents to the ED with chest pain and
suspected acute coronary syndrome (e.g., retrosternal,
diaphoresis, nausea, and/or vomiting) should be given _
as soon as possible.

A) aspirin
B) heparin
C) ibuprofen
Q60 Cardiology
A patient who presents to the ED with chest pain and
suspected acute coronary syndrome (e.g., retrosternal,
diaphoresis, nausea, and/or vomiting) should be given _
as soon as possible.

A) aspirin
B) heparin
C) ibuprofen
Q60 Cardiology
A patient who presents to the ED with chest pain and
suspected acute coronary syndrome (e.g., retrosternal,
diaphoresis, nausea, and/or vomiting) should be given _
as soon as possible.
A) aspirin - give even if ACS just SUSPECTED - aspirin
inhibits TXA2, leading to significant antiplatelet effects;
B) heparin - if positive cardiac markers (or in PE)
Q61 Cardiology
Infants of diabetic mother who develop transient
hypertrophic cardiomyopathy should be treated with:

A) Propranolol
B) Indomethacin
Q61 Cardiology
Infants of diabetic mother who develop transient
hypertrophic cardiomyopathy should be treated with:

A) Propranolol
B) Indomethacin
Q61 Cardiology
Infants of diabetic mother who develop transient
hypertrophic cardiomyopathy should be treated with:

Propranolol - reduces LVOT obstruction by


increasing LV diastolic filling time and EDV due to
decreased HR
Q62 Cardiology
What value of the ABI supports a diagnosis of PAD?

A) less than .9
B) more than .9
Q62 Cardiology
What value of the ABI supports a diagnosis of PAD?

less than .9 = ABI


more than 1.3 = suggests
calcified & uncompressible vessels
Q63 Cardiology
A person running a bbq develops symptoms of MI;
cardiac troponin I levels are elevated. Cath shows
no acute occlusion of the coronary arteries. What is
the next step?

A) Carboxyhemoglobin level
B) CT angiogram
Q63 Cardiology
A person running a bbq develops symptoms of MI;
cardiac troponin I levels are elevated. Cath shows
no acute occlusion of the coronary arteries. What is
the next step?

A) Carboxyhemoglobin level
B) CT angiogram
Q63 Cardiology
A person running a bbq develops symptoms of MI;
cardiac troponin I levels are elevated. Cath shows
no acute occlusion of the coronary arteries. What is
the next step?

A) Carboxyhemoglobin level - CO poisoning can


lead to acute MI (30% of cases); tx: 100% O2
B) CT angiogram
Q64 Cardiology
A patient with new intermittent claudication should
be given aspirin, lipid-lowering therapy, as well as:

A) recommendation for exercise


B) pentoxifylline
C) cilostazol
Q64 Cardiology
A patient with new intermittent claudication should
be given aspirin, lipid-lowering therapy, as well as:

A) recommendation for exercise


B) pentoxifylline
C) cilostazol
Q64 Cardiology
A patient with new intermittent claudication should
be given aspirin, lipid-lowering therapy, as well as:

recommendation for exercise - at least 30 minutes


3x a week, for a minimum of 12 weeks; improves
outcomes
Q65 Cardiology
Which of the following post-MI complications occurs
several months after an MI?

A) IV septum rupture
B) Papillary muscle rupture
C) Free wall rupture
D) LV aneurysm
Q65 Cardiology
Which of the following post-MI complications occurs
several months after an MI?

A) IV septum rupture
B) Papillary muscle rupture
C) Free wall rupture
D) LV aneurysm
Q65 Cardiology
Which of the following post-MI complications occurs
several months after an MI?

LV aneurysm - results from scarring and fibrosis of a


large portion of the LV wall; EKG findings include
persistent ST elevation and deep Q waves in the
same leads; can cause HF, ventricular arrhythmias;
diagnose with echo
Q66 Cardiology
Acute severe chest pain in a woman with short
stature and primary amenorrhea should raise
concern for:

A) aortic dissection
B) pulmonary embolism
Q66 Cardiology
Acute severe chest pain in a woman with short
stature and primary amenorrhea should raise
concern for:

A) aortic dissection
B) pulmonary embolism
Q66 Cardiology
Turner syndrome - genetic condition caused by
partial/complete loss of an X chromosome; short
stature (since X chromosome has genes for growth
in females); webbed neck; gonadal dysgenesis;
POI; broad chest, micrognathia, scoliosis, aortic
dissection or coarc. (TS patients should get regular
screening with echo or MRI).
Q67 Cardiology
A 58-year-old man undergoes a cardiac cath procedure, and
several hours later develops back pain and a BP = 76/59. Heart
sounds are normal and chest is clear to auscultation. Neck
veins are flat. 1000mL of normal saline improves symptoms
with a slight improvement in BP. What is the next step?

A) CT scan of abdomen/pelvis
B) Transthoracic echo
Q67 Cardiology
A 58-year-old man undergoes a cardiac cath procedure, and
several hours later develops back pain and a BP = 76/59.
Heart sounds are normal and chest is clear to auscultation.
Neck veins are flat. 1000mL of normal saline improves
symptoms with a slight improvement in BP. What is the next
step?

A) CT scan of abdomen/pelvis
B) Transthoracic echo
Q67 Cardiology
*cardiac cath can lead to retroperitoneal hematoma due to
bleeding from arterial access site; confirm diagnosis with CT or
ultrasound; often seen in those on anticoagulation; sudden
onset of hypotension, tachycardia, and flat neck veins;
treatment is usually supportive*
Q68 Cardiology
Caution during elective surgery for patients with
hypertrophic cardiomyopathy involves:

A) adequate hydration
B) avoiding beta blockers
Q68 Cardiology
Caution during elective surgery for patients with
hypertrophic cardiomyopathy involves:

A) adequate hydration
B) avoiding beta blockers
Q68 Cardiology
Caution during elective surgery for patients with
hypertrophic cardiomyopathy involves adequate
hydration; this is because patients with HCM have
LVOT that can potentially severely limit CO; low LV
blood volume worsens the obstruction; that's why
high LV blood volume is needed thoughout the
cardiac cycle
Q69 Cardiology
Which anti-hypertensive medicationis first line in
patients with CKD?

A) ACE's/ARBS
B) Beta blockers
C) CCB's
D) Diuretics (thiazides)
Q69 Cardiology
Which anti-hypertensive medication is first line in
patients with CKD?

A) ACE's/ARBS
B) Beta blockers
C) CCB's
D) Diuretics (thiazides)
Q69 Cardiology
Which anti-hypertensive medication is first line in
patients with CKD?

A) ACE's/ARBS - nephroprotective
B) Beta blockers
C) CCB's
D) Diuretics (thiazides) - not for CKD!
Q69 Cardiology
Which anti-hypertensive medication is first line in
patients with CKD?

A) ACE's/ARBS - nephroprotective
(in pregnancy, methyldopa is used, as well as
B) Beta blockers labetalol or hydralazine)
(in BPH, use diuretics and alpha blockers)
C) CCB's
D) Diuretics (thiazides) - not for CKD!
Q70 Cardiology
A dehydrated 27-year-old woman is brought to the
ED due to confusion; BP = 81/32. EKG shows no P
waves and a sine wave pattern. What is the next
best step?

A) Atropine
B) Calcium gluconate
Q70 Cardiology
A dehydrated 27-year-old woman is brought to the
ED due to confusion; BP = 81/32. EKG shows no P
waves and a sine wave pattern. What is the next
best step?

A) Atropine
B) Calcium gluconate
Q70 Cardiology
Causes of Hyperkalemia (K+ > 5mEq/L)
-spuriously (e.g., clenched fists during blood draws)
-by drugs (spironolactone, amiloride, ACEI's, trimethoprim,
beta blockers, NSAIDs),
-renal insufficiency, type 4 RA
-cell lysis, tumor lysis syndrome
-food (e.g., most fruits)
Q70 Cardiology
**always give calcium gluconate to protect
cardiac cell membrane if K+ > 6.5 or EKG
changes!!!**
Q70 Cardiology
C BIG K = hyperkalemia treatment
C = calcium
B = beta agonist (e.g., albuterol)
I = insulin +
G = glucose
K = kayexalate (falling out of favor)
Q71 Cardiology
In hypovolemic shock, SVR is:

A) increased
B) decreased
Q71 Cardiology
In hypovolemic shock, SVR is:

A) increased
B) decreased
Q71 Cardiology
In hypovolemic shock, SVR is:

A) increased - to compensate for low CO


B) decreased
Q72 Cardiology
A systolic murmur at the lower sternal border that
increases with inspiration is:

A) MR
B) TR
Q72 Cardiology
A systolic murmur at the lower sternal border that
increases with inspiration is:

A) MR - holosystolic; best at apex; does NOT


increase with inspiration
B) TR - increases with inspiration
due to increased venous return to the right heart
Q72 Cardiology
Q73 Cardiology
A 37-year-old woman with a history of fevers and weight loss
presents to the ED with acute onset, left sided weakness. Brain
imaging shows several small infarcts in the parts of the right lobes.
Echo shows a mass in the left atrium. What is the diagnosis?

A) A tumor
B) Valve degeneration
C) Thrombus
Q73 Cardiology
A 37-year-old woman with a history of fevers and weight loss
presents to the ED with acute onset, left sided weakness. Brain
imaging shows severeal small infarcts in the parts of the right
lobes. Echo shows a mass in the left atrium. What is the
diagnosis?

A) A tumor
B) Valve degeneration
C) Thrombus
Q73 Cardiology
A 37-year-old woman with a history of fevers and weight loss
presents to the ED with acute onset, left sided weakness. Brain
imaging shows several small infarcts in the parts of the right lobes.
Echo shows a mass in the left atrium. What is the diagnosi small
infarcts in the parts of the right lobes. Echo shows a mass in the
left atrium. What is the diagnosis?s?

A) A tumor - myxomas are benign tumors and are the most


common primary cardiac neoplasm; 80% arise from the LA;
fragments of the myxoma can embolize, leading to stroke or
limb/mesenteric ischemia; murmur mimicks mitral stenosis
diagnose with echo; tx with prompt surgical resection
Q74 Cardiology
A patient with a history of tetralogy of Fallot has a
decrescendo diastolic murmur heard at the left sternal
border; the murmur increases with inspiration. What is
the diagnosis?

A) Tricuspid regurgitation
B) Pulmonic regurgitation
Q74 Cardiology
A patient with a history of tetralogy of Fallot has a
decrescendo diastolic murmur heard at the left sternal
border; the murmur increases with inspiration. What is
the diagnosis?

A) Tricuspid regurgitation
B) Pulmonic regurgitation - a common complication of
tetralogy of Fallot repair
Q75 Cardiology
A patient presents with suspected prosthetic valve
endocarditis. Broad spectrum antibiotics are started after
cultures grow VRSA. Transthoracic echo shows no
abnormalities in the valves. What is the next step?

A) IV vancomycin
B) Transesophageal echo
Q75 Cardiology
A patient presents with suspected prosthetic valve
endocarditis. Broad spectrum antibiotics are started after
cultures grow VRSA. Transthoracic echo shows no
abnormalities in the valves. What is the next step?

A) IV vancomycin
B) Transesophageal echo
Q75 Cardiology
A patient presents with suspected prosthetic valve
endocarditis. Broad spectrum antibiotics are started after
cultures grow VRSA. Transthoracic echo shows no
abnormalities in the valves. What is the next step?

A) IV vancomycin
B) Transesophageal echo - transthoracic is unreliable
(detects PVE less than 60% of the time); get TEE
Q76 Cardiology
Besides exercise, smoking cessation, and blood
pressure control, what should be done for PAD (i.e.,
intermittent claudication and ABI < .9)?

A) Take aspirin and atorvastatin


B) Warfarin
Q76 Cardiology
Besides exercise, smoking cessation, and blood
pressure control, what should be done for PAD (i.e.,
intermittent claudication and ABI < .9)?

A) Take aspirin and atorvastatin


B) Warfarin
Q77 Cardiology
A 6-year-old boy has a murmur that looks like this.
What is the diagnosis?

A) PDA
B) Aortic stenosis
Q77 Cardiology
A 6-year-old boy has a murmur that looks like this.
What is the diagnosis?

A) PDA
B) Aortic stenosis
Q77 Cardiology
PDA - continuous murmur with maximal intensity at S2;
murmur due to turbulent blood flow from aorta to pulmonary;
best heard at left infraclavicular or posterior interscapular
region; small PDA's are asymptomatic, larger ones can lead
to ventricular heart failure due to LV compensation for the
significant reduction in SVR created by the PDA shunt
Q78 Cardiology
How does hypokalemia affect the EKG?

A) U waves, T-wave flattening


B) no changes
Q78 Cardiology
How does hypokalemia affect the EKG?

A) U waves, T-wave flattening


B) no changes
Q78 Cardiology
Hypokalemia
-e.g., due to insulin, b2 agonists, diarrhea, laxatives,
vomiting, loop diuretics, Bartter & Gitelman syndrome,
drugs (e.g., gentamicin), diabetic ketoacidosis
-presents with fatigue, weakness, cramps, paresthesias;
EKG may show T wave flattening and U waves (an
additional wave after T wave)
Q79 Cardiology
A 35-year-old man presents for a routine visit. BP =
180/106. Bilateral, nontender upper abdominal masses
are palpated. What is the next step?

A) Ultrasound of the abdomen


B) Urine metanephrines
Q79 Cardiology
A 35-year-old man presents for a routine visit. BP =
180/106. Bilateral, nontender upper abdominal masses
are palpated. What is the next step?

A) Ultrasound of the abdomen


B) Urine metanephrines
Q79 Cardiology
Early onset hypertension and bilateral upper abdominal
masses - most likely ADPKD, the most common genetic
cause of CKD. Leads to cystic degeneration of the renal
parenchyma; HTN is a common early disease
manifestations (ages 30-40); extrarenal complications of
ADPKD include cerebral aneurysms, hepatic/pancreatic
cysts, colonic diverticula, valvular abnormalities
Q80 Cardiology
Medications which are given after an MI include: beta
blockers, ACE inhibitors (or ARBS), statins, and:

A) aspirin and P2Y12 receptor blocker


B) aspirin and nitroglycerin
Q80 Cardiology
Medications which are given after an MI include: beta
blockers, ACE inhibitors (or ARBS), statins, and:

A) aspirin and P2Y12 receptor blocker


B) aspirin and nitroglycerin
Q80 Cardiology
Medications which are given after an MI include: beta blockers,
ACE inhibitors (or ARBS), statins, and:
-beta blockers - decrease oxygen demand; start within 24
hours
-ACE inhibitors - improve LVEF - best for anterior MI's
-statins - reduce risk of another MI; goal LDL < 70
-dual antiplatelet therapy- (aspirin indefinitely and second
antiplatelet drug for 6-24 months); prevents another MI by
preventing clots from forming
Q81 Cardiology
Metabolic acidosis, neurologic changes (e.g., agitation,
seizures), and tachycardia is seen with:

A) Acute cyanide toxicity


B) Arsenic poisoning
Q81 Cardiology
Metabolic acidosis, neurologic changes (e.g., agitation,
seizures), and tachycardia is seen with:

A) Acute cyanide toxicity


B) Arsenic poisoning
Q81 Cardiology
ACUTE CYANIDE TOXICITY
ACUTE CYANIDE TOXICITY IS a side effect of
nitroprusside (because metabolism of nitroprusside
releases NO which induces vasodilation, as well as
cyanide ions; thus, cyanide toxicity can occur, esp. in
patients with renal insufficiency; oxidative
phosphorylation inhibition; confusion, agitation, seizures,
eventual cardiovascular collapse)
Q82 Cardiology
A healthy 2-day-old baby has acrocyanosis. What test
should be done?

A) Echocardiogram
B) Pulse oximetry
Q82 Cardiology
A healthy 2-day-old baby has acrocyanosis. What test
should be done?

A) Echocardiogram
B) Pulse oximetry
Q83 Cardiology
Hypertrophic cardiomyopathy differs from athlete's heart
in that in HCM, the left atrium is:

A) enlarged
B) normal
Q83 Cardiology
Hypertrophic cardiomyopathy differs from athlete's heart
in that in HCM, the left atrium is:

A) enlarged
B) normal
Q83 Cardiology
Q84 Cardiology
A heart medication that acts to stimulate beta-1
adrenergic receptors leads to:

A) Decreased LV end-systolic volume


B) Increased cardiac preload
Q84 Cardiology
A heart medication that acts to stimulate beta-1
adrenergic receptors leads to:

A) Decreased LV end-systolic volume


B) Increased cardiac preload
Q84 Cardiology
A heart medication that acts to stimulate beta-1
adrenergic receptors leads to:

A) Decreased LV end-systolic volume - dobutamine!


increases contractility; increases CO!
B) Increased cardiac preload - no effect
Q85 Cardiology
Low set ears, micrognathia, cleft palate, absent thymus,
and _.

A) Hypocalcemia
B) Duodenal atresia
Q85 Cardiology
Low set ears, micrognathia, cleft palate, absent thymus,
and _.

A) Hypocalcemia
B) Duodenal atresia
Q85 Cardiology
Low set ears, micrognathia, cleft palate, absent thymus,
and _.

A) Hypocalcemia
B) Duodenal atresia
Q86 Cardiology
Bilateral thigh claudication, absent femoral pulses, and _.

A) impotence
B) headache
Q86 Cardiology
Bilateral thigh claudication, absent femoral pulses, and _.

A) impotence
B) headache
Q86 Cardiology
Bilateral thigh claudication, absent femoral pulses, and _.

A) impotence
B) headache

Leriche syndrome! (most common in men with a


predisposition for atherosclerosis)
Q87 Cardiology
Respiratory failure due to acute decompensated HF is
managed with:

A) Noninvasive positive pressure ventilation


B) Dobutamine
Q87 Cardiology
Respiratory failure due to acute decompensated HF is
managed with:

A) Noninvasive positive pressure ventilation


B) Dobutamine
Q87 Cardiology
Respiratory failure due to acute decompensated HF is
managed with:

A) Noninvasive positive pressure ventilation - by


intubation if needed (also, IV diuretics and vasodilators to
reduce preload)
B) Dobutamine - increases CO, only for patients in
cardiogenic shock
Q88 Cardiology
An exaggerated fall in systemic blood pressure > 10
mmHg can be seen in cardiac tamponade and _ .

A) Severe asthma
B) Aortic regurgitation
Q88 Cardiology
An exaggerated fall in systemic blood pressure > 10
mmHg upon inspiration can be seen in cardiac
tamponade and _ .

A) Severe asthma
B) Aortic regurgitation
Q89 Cardiology
Edema + ascites + elevated JVP + clear lungs + low
voltage QRS complexes =

A) constrictive pericarditis
B) amyloidosis
Q89 Cardiology
Edema + ascites + elevated JVP + clear lungs + low
voltage QRS complexes =

A) constrictive pericarditis
B) amyloidosis
Q89 Cardiology
Edema + ascites + elevated JVP + clear lungs + low
voltage QRS complexes =

A) constrictive pericarditis - results from thickened and


often calcified pericardium; limits diastolic filling;
manifestations similar to right sided heart failure
pericardiectomy is the only definitive treatment
Q90 Cardiology
A man develops acute limb ischemia secondary to a left
atrial thrombus. This could have been best prevented
with:

A) Clopidogrel
B) Aspirin
C) Cilostazol
D) Apixaban
Q90 Cardiology
A man develops acute limb ischemia secondary to a left
atrial thrombus. This could have been best prevented
with:

A) Clopidogrel
B) Aspirin
C) Cilostazol
D) Apixaban
Q90 Cardiology
A man develops acute limb ischemia secondary to a left
atrial thrombus. This could have been best prevented
with:

A) Clopidogrel - not as good as a DOAC


B) Aspirin - not as good as a DOAC
C) Cilostazol - for intermittent claudication
D) Apixaban - a direct oral anticoagulant (DOAC)
Q91 Cardiology
Aortic valve infective endocarditis causes _ in 1/3 cases,
a complication which causes conduction abnormalities.

A) Perivalvular abscess
B) Mitral valve perforation
Q91 Cardiology
Aortic valve infective endocarditis causes _ in 1/3 cases,
a complication which causes conduction abnormalities.

A) Perivalvular abscess
B) Mitral valve perforation
Q91 Cardiology
Aortic valve infective endocarditis causes _ in 1/3 cases,
a complication which causes conduction abnormalities.

A) Perivalvular abscess - suspect in patients with


persistent bacteremia or cardiac conduction
abnormalities, such as heart block. 18% mortality!
B) Mitral valve perforation
Q92 Cardiology
A man presents with pleuritic chest pain, dyspnea,
tachypnea, and tachycardia, although CXR is normal.
What is the diagnosis?

A) Pulmonary embolism
B) Acute MI
Q92 Cardiology
A man presents with pleuritic chest pain, dyspnea,
tachypnea, and tachycardia, although CXR is normal.
What is the diagnosis?

A) Pulmonary embolism
B) Acute MI
Q92 Cardiology
A man presents with pleuritic chest pain, dyspnea,
tachypnea, and tachycardia, although CXR is normal.
What is the diagnosis?

A) Pulmonary embolism - remember! Pulmonary


embolism CXR may be normal! (and for that matter,
some PATIENTS are completely asymptomatic!!)
B) Acute MI
Q93 Cardiology
A baby with poor feeding presents with a holosystolic
murmur that is loudest at the left lower sternal border and
an apical diastolic rumble. What is the diagnosis?

A) VSD
B) Tetralogy of fallot
Q93 Cardiology
A baby with poor feeding presents with a holosystolic
murmur that is loudest at the left lower sternal border and
an apical diastolic rumble. What is the diagnosis?

A) VSD
B) Tetralogy of fallot
Q93 Cardiology
A baby with poor feeding presents with a holosystolic
murmur that is loudest at the left lower sternal border and
an apical diastolic rumble. What is the diagnosis?

A) VSD - diastolic rumble due to increased blood flow


across the mitral valve
B) Tetralogy of fallot - systolic ejection murmur
Q94 Cardiology
Nitrates (e.g. nitroprusside) should not be taken
with Viagra. T/F.

A) True
B) False
Q94 Cardiology
Nitrates (e.g. nitroprusside) should not be taken
with Viagra. T/F.

A) True
B) False
Q94 Cardiology
Nitrates (e.g. nitroprusside) should not be taken
with Viagra. T/F.

A) True
B) False
How long does one need to wait after an MI to re-engage in sexual activity?
On average 4 weeks, depending on symptoms; an inferior MI requires a
shorter wait than an anterior MI
Q95 Cardiology
Easy bruising, proteinuria, and restrictive cardiomyopathy can be
seen in _.

A) amyloidosis
B) hemochromatosis
C) sarcoidosis
Q95 Cardiology
Easy bruising, proteinuria, and restrictive cardiomyopathy can be
seen in _.

A) amyloidosis
B) hemochromatosis
C) sarcoidosis
Q95 Cardiology
Easy bruising, proteinuria, and restrictive cardiomyopathy can be
seen in _.

amyloidosis - involves deposition of insoluble protein fibrils in


various tissues throughout the body. cardiac involvement is
common (esp. restrictive cardiomyopathy); amyloid deposits in
the skin leads to easy bruising; amyloid deposits in the kidney
leads to proteinuria
Q96 Cardiology
In anaphylactic shock, PCWP is _.

A) increased
B) decreased
Q96 Cardiology
In anaphylactic shock, PCWP is _.

A) increased
B) decreased
Q96 Cardiology
In anaphylactic shock, PCWP is _.

A) increased
B) decreased - arteriolar vasodilation decreased SVR; venular
vasodilation decreases central venous pressure and PCWP
Q97 Cardiology
A 60-year-old woman, who had been told several days ago that
her husband died, has mid-sternal chest pain and nausea; EKG
shows T wave inversions, but no obstructive coronary artery
disease. What is the diagnosis?

A) Takotsubo cardiomyopathy
B) Panic disorder
Q97 Cardiology
A 60-year-old woman, who had been told several days ago that
her husband died, has mid-sternal chest pain and nausea; EKG
shows T wave inversions, but no obstructive coronary artery
disease. What is the diagnosis?

A) Takotsubo cardiomyopathy
B) Panic disorder
Q97 Cardiology

Takotsubo cardiomyopathy - "stress-induced cardiomyopathy";


mostly seen in post-menopausal women; characterized by LV
"ballooning"; EKG shows evidence of ischemia (e.g., ST
elevation, T wave inversions) chest pain mimics MI; treatment is
supportive, usually resolves in a few weeks
Q98 Cardiology
A 12-year-old boy with a history of polyarthritis and febrile illness
now presents with mitral regurgitation on echo. What medication
should be given?

A) penicillin
B) no medication
Q98 Cardiology
A 12-year-old boy with a history of polyarthritis and febrile illness
now presents with mitral regurgitation on echo. What medication
should be given?

A) penicillin
B) no medication
Q98 Cardiology
A 12-year-old boy with a history of polyarthritis and febrile illness
now presents with mitral regurgitation on echo. What medication
should be given?

A) penicillin - since presentation is suggestive of rheumatic heart


disease, a complication of GAS, penicillin prophylaxis should be
given to prevent recurrence; also, monitor every 6-12 month for
signs of LV dysfunction
B) no medication
Q99 Cardiology
Which medication can safely be taken during pregnancy?

A) Spironolactone
B) Chlorthalidone
C) Atenolol
D) Lisinopril
E) Valsartan
F) Nifedipine
Q99 Cardiology
Which medication can safely be taken during pregnancy?

A) Spironolactone
B) Chlorthalidone
C) Atenolol
D) Lisinopril
E) Valsartan
F) Nifedipine
Q99 Cardiology
Which medication can safely be taken during pregnancy?

A) Spironolactone - antiandrogen on male fetus


B) Chlorthalidone - decreased maternal blood volume
C) Atenolol - lower birth weight (other BB's okay)
D) Lisinopril - fetal kidney dysfunction
E) Valsartan - fetal kidney dysfunction
F) Nifedipine - also labetolol, hydralazine, and methyldopa
Q100 Cardiology
After insufflation of CO2 gas into the abdominal cavity, a patient
develops bradycardia and AV block. What is the mechanism?

A) stretching of peritoneum
B) systemic CO2 absorption
Q100 Cardiology
After insufflation of CO2 gas into the abdominal cavity, a patient
develops bradycardia and AV block. What is the mechanism?

A) stretching of peritoneum
B) systemic CO2 absorption
Q100 Cardiology
After insufflation of CO2 gas into the abdominal cavity, a patient
develops bradycardia and AV block. What is the mechanism?

stretching of peritoneum - CO2 insufflation during laparoscopic


surgery (e.g., appendectomy) stimulates stretch receptors on the
the peritoneum; increase in vagal tone; patients can develop
severe bradycardia, AV block, and sometimes even asystole
Q101 Cardiology
A 33-year-old woman presents with chest pain for the last 4
months. The pain is describes as a squeezing type pain that
radiates to the neck and can last up to 3 hours. BP = 120/81.
EKG and exercise stress test are negative. What is the most
likely cause of the pain?

A) CAD
B) Esophageal disease
Q101 Cardiology
A 33-year-old woman presents with chest pain for the last 4
months. The pain is describes as a squeezing type pain that
radiates to the neck and can last up to 3 hours. BP = 120/81.
EKG and exercise stress test are negative. What is the most
likely cause of the pain?

A) CAD
B) Esophageal disease
Q101 Cardiology

Esophageal disease - esophageal chest pain, e.g., GERD, is the


most common condition confused with angina; but distinguishing
features include: esophageal pain lasts a lot longer (e.g., hours),
whereas angina lasts minutes; esophageal chest pain comes on
with meals or position, angina with exertion
Q102 Cardiology
A 2 hours old baby, after a delivery complicated by thick
meconium, now has severe respiratory distress; the baby has a
postductal oxygen saturation in the right foot significantly lower
(78%) than the preductal oxygen in the right hand (91%). What is
the next step?

A) albuterol
B) nitric oxide
Q102 Cardiology
A 2 hours old baby, after a delivery complicated by thick
meconium, now has severe respiratory distress; the baby has a
postductal oxygen saturation in the right foot significantly lower
(78%) than the preductal oxygen in the right hand (91%). What is
the next step?

A) albuterol
B) nitric oxide
Q102 Cardiology
nitric oxide - oxygen saturation in foot lower than in hand in the
setting of meconium aspiration indicates pulmonary hypertension
of the newborn (i.e., inappropriately persistent PVR elevation with
R to L shunting across the ductus arteriosus, creating a
discrepancy between preductal and postductal oxygen
saturations; associated with lung injury due to meconium
aspiration); treatment: reduction in PVR through
oxygenation/ventilation and nitric oxide to dilate pulmonary
vasculature, to decrease R to L shunting
Q103 Cardiology
A 56-year-old man with a BP = 176/105 despite a three-drug anti-
hypertensive regimen also has systolic-diastolic upper abdominal
bruit. What is the most likely diagnosis?

A) Renal artery stenosis


B) Aortic coarctation
C) Abdominal aortic aneurysm
Q103 Cardiology
A 56-year-old man with a BP = 176/105 despite a three-drug anti-
hypertensive regimen also has systolic-diastolic upper abdominal
bruit. What is the most likely diagnosis?

A) Renal artery stenosis


B) Aortic coarctation
C) Abdominal aortic aneurysm
Q103 Cardiology
A 56-year-old man with a BP = 176/105 despite a three-drug anti-
hypertensive regimen also has systolic-diastolic upper abdominal
bruit. What is the most likely diagnosis?

Renal artery stenosis - the most common and correctable cuase


of secondary hypertension; seen in widespread atherosclerotic
disease and resistant hypertension; abdominal bruit heard in
about 40% of patients (diagnosis: renal Doppler ultrasound, CT or
MR angiography)
Q104 Cardiology
A 34-year-old woman from a developing country presents with
left-sided weakness that started several hours earlier. She has
had periods of palpitations and an irregular heartbeat, along with
dyspnea, nightime cough, and hemoptysis. What is the
diagnosis?

A) Mitral stenosis
B) PAH
Q104 Cardiology
A 34-year-old woman from a developing country presents with
left-sided weakness that started several hours earlier. She has
had periods of palpitations and an irregular heartbeat, along with
dyspnea, nightime cough, and hemoptysis. What is the
diagnosis?

A) Mitral stenosis
B) PAH
Q105 Cardiology
A crescendo-decrescendo systolic murmur heard best
along the left sternal border in a 25-year old woman is
most likely due to:

A) bicuspid aortic valve


B) interventricular septal hypertrophy
Q105 Cardiology
A crescendo-decrescendo systolic murmur heard best
along the left sternal border in a 25-year old woman is
most likely due to:

A) bicuspid aortic valve


B) interventricular septal hypertrophy
Q106 Cardiology
Uveitis and complete AV block are seen in:

A) Sarcoidosis
B) Pneumoconiosis
Q106 Cardiology
Uveitis and complete AV block are seen in:

A) Sarcoidosis
B) Pneumoconiosis
Q106 Cardiology
Uveitis and complete AV block are seen in:

A) Sarcoidosis
Cardiac sarcoidosis is a condition in which noncaseating
granulomas infiltrate the myocardium potentially resulting
in AV block, arrhythmia, cardiomyopathy, HF, and sudden
cardiac death.
B) Pneumoconiosis
Q107 Cardiology
Which of the following conditions is a common cause of
dilated cardiomyopathy in young adults?

A) Virus infection
B) Coccidioides
C) Coronary atherosclerosis
Q107 Cardiology
Which of the following conditions is a common cause of
dilated cardiomyopathy in young adults?

A) Virus infection
B) Coccidioides
C) Coronary atherosclerosis
Q107 Cardiology

Several viruses have been implicated in DCM (e.g.,


parvovirus, HHV-6, adenovirus, coxsackievirus, influenza,
and HIV); it is thought that the virus invades cardiac
myocytes, leading to cell damage and impaired
contractility, and then ventricular dilation; patients
commonly present with symptoms of decompensated HF
(e.g., dyspnea, orthopnea, edema)
Q108 Cardiology
What is the most common congenital heart defect in Down
syndrome patients?

A) ASD
B) PDA
C) Coarctation of the aorta
Q108 Cardiology
What is the most common congenital heart defect in Down
syndrome patients?

A) ASD
B) PDA
C) Coarctation of the aorta
Q108 Cardiology

A) ASD - complete ASD; clinical features include HF in


infancy, a fixed split S2, and a systolic ejection murmur as
increased blood flows across the pulmonary valve
B) PDA - congenital Rubella
C) Coarctation of the aorta - Turner syndrome
Q109 Cardiology
A patient presents with a blood pressure of 174/75. This is
most probably caused by:

A) increased intravascular volume


B) increased renin
C) decreased compliance of the arterial walls
Q109 Cardiology
A patient presents with a blood pressure of 174/75. This is
most probably caused by:

A) increased intravascular volume


B) increased renin
C) decreased compliance of the arterial walls
Q109 Cardiology
A patient presents with a blood pressure of 174/75. This is
most probably caused by:

A) increased intravascular volume - (increases both systolic and diastolic)


B) increased renin - renal artery stenosis (increase in both systolic and
diastolic)
C) decreased compliance of the arterial walls - isolated systolic hypertension
(~sys. > 140; ~dia. < 90) is most common in elderly patients, and is due to
increased rigidity of the aortic/arterial walls, leading to increased systolic blood
pressure; treat with CCB's and thiazides
Q110 Cardiology
Do patients with prosthetic heart valves require
antimicrobial prophylaxis (against infective
endocarditis) before undergoing colonoscopy?

A) yes
B) no
Q110 Cardiology
Do patients with prosthetic heart valves require
antimicrobial prophylaxis (against infective
endocarditis) before undergoing colonoscopy?

A) yes
B) no
Q110 Cardiology
Do patients with prosthetic heart valves require
antimicrobial prophylaxis (against infective
endocarditis) before undergoing colonoscopy?
A) yes
B) no - prophylaxis required for invasive procedures involving the mouth,
respiratory mucosa, infected tissues, and infected areas of the GI/GU
tract (NOT for most other GI/GU procedures)
Q111 Cardiology
A young patient with chronic back pain, intermittent
bilateral heel pain, impaired spinal mobility, limited
chest expansion, is most likely to have which
cardiac defect:

A) mitral valve prolapse


B) aortic regurgitation
Q111 Cardiology
A young patient with chronic back pain, intermittent
bilateral heel pain, impaired spinal mobility, limited
chest expansion, is most likely to have which
cardiac defect:

A) mitral valve prolapse


B) aortic regurgitation
Q111 Cardiology
Ankylosing spondylitis is an inflammatory
arthropathy involving chronic back pain, impaired
spinal mobility and enthesitis; aortic regurgitation
is a complication (10%) of ankylosing spondylitis
(mitral and tricuspid valves are unaffected); may
cause conduction abnormalities
Q112 Cardiology
Brugada syndrome is due to a structural defect in
the heart. T/F.

A) True
B) False
Q112 Cardiology
Brugada syndrome is due to a structural defect in
the heart. T/F.

A) True
B) False
Q112 Cardiology
Brugada syndrome is due to a structural defect in
the heart. T/F.
False - it is an autosomal dominant condition affecting
cardiac SODIUM CHANNELS. Patients have EKG
abnormalities (e.g., ST elevations in leads V1-V2), but no
structural abnormalities are typically present; may die
from ventricular arrhythmia
Q113 Cardiology
Does this EKG indicate a right or left BBB?

A) R
B) L
Q113 Cardiology
Does this EKG indicate a right or left BBB?

A) R
B) L
Q113 Cardiology
Does this EKG indicate a right or left BBB?

A) R
B) L
Q113 Cardiology
Does this EKG indicate a right or left BBB?

A) R
B) L
LEFT = WiLLiaM (W & M)
Q114 Cardiology
A 43-year-old woman who has smoked for 20 years has nocturnal
substernal chest pain episodes that last about 10 minutes and
then resolve. She reports no exertional chest pain or discomfort.
Upon exam, ST elevations are seen during a pain episode. What
is the pathophysiology of this condition?

A) occlusion of small penetrating arteries


B) vascular smooth muscle hyperreactivity
Q114 Cardiology
A 43-year-old woman who has smoked for 20 years has nocturnal
substernal chest pain episodes that last about 10 minutes and
then resolve. She reports no exertional chest pain or discomfort.
Upon exam, ST elevations are seen during a pain episode. What
is the pathophysiology of this condition?

A) occlusion of small penetrating arteries


B) vascular smooth muscle hyperreactivity
Q114 Cardiology
A 43-year-old woman who has smoked for 20 years has nocturnal
substernal chest pain episodes that last about 10 minutes and
then resolve. She reports no exertional chest pain or discomfort.
Upon exam, ST elevations are seen during a pain episode. What
is the pathophysiology of this condition?

A) occlusion of small penetrating arteries


B) vascular smooth muscle hyperreactivity - vasospastic angina
Q114 Cardiology

Vasospastic angina (previously Prinzmetal angina) is characterized


by recurrent episodes chest discomfort resulting from hyperreactivity
of vascular smooth muscle, leading to intermittent vasospasm of the
coronary artery. Typically treated with CCB's.
Q115 Cardiology
Abrupt-onset chest pain and mediastinal
widening (as seen in the image) is seen in:

A) aortic dissection
B) pulmonary embolism
Q115 Cardiology
Abrupt-onset chest pain and mediastinal
widening (as seen in the image) is seen in:

A) aortic dissection
B) pulmonary embolism
Q115 Cardiology
A) aortic dissection - Aortic dissection commonly presents with
sudden chest pain; mediastinal widening can be seen on
chest x-ray; CT scan is usually the diagnostic test of choice for
suspected AD; treat with beta blockers (to decrease "shearing
forces" that worsen the dissection), then nitroprusside, then
usually (>90%) surgical correction
B) pulmonary embolism - the mediastinum is ENCLOSED by
the pleura
Q116 Cardiology
An asymptomatic 1-day old boy shows a pulse ox = 99% in
the right hand and 89% in the right foot. Distal pulses are
strong. What is the diagnosis?

A) Coarctation of the aorta


B) Fetal circulation persistence
Q116 Cardiology
An asymptomatic 1-day old boy shows a pulse ox = 99% in
the right hand and 89% in the right foot. Distal pulses are
strong. What is the diagnosis?

A) Coarctation of the aorta


B) Fetal circulation persistence
Q116 Cardiology
An asymptomatic 1-day old boy shows a pulse ox = 99% in
the right hand and 89% in the right foot. Distal pulses are
strong. What is the diagnosis?
A) Coarctation of the aorta - also leads to R to L shunting
across the PDA, but these patients have diminished femoral
pulses due to hypoplasia of the aortic arch
B) Fetal circulation persistence - abnormal persistence of
elevated pulmonary vascular resistance, leading to R to L
shunting across a PDA
Q117 Cardiology
An unresponsive patient has no palpable pulse; the cardiac
monitor still shows atrial fibrillation. What is the next best step?

A) Cardioversion
B) Defibrillation
C) Compressions
Q117 Cardiology
An unresponsive patient has no palpable pulse; the cardiac
monitor still shows atrial fibrillation. What is the next best step?

A) Cardioversion
B) Defibrillation
C) Compressions
Q117 Cardiology
An unresponsive patient has no palpable pulse; the cardiac
monitor still shows atrial fibrillation. What is the next best step?

Compressions - pulseless electrical activity (PEA) or asystole


should be managed with uninterrupted CPR (along with
vasopressor therapy if available) to maintain cerebral and
coronary perfusion; defibrillation or cardioversion do not work in
these patients
Q118 Cardiology
A 2-year-old boy is brought to the ED with fever and respiratory
distress. He recovered from a URI (rhinorrhea and congestion)
about a week ago, but fever and fatigue has persisted. Cardiac
exam shows tachycardia, an S3 gallop, and a holosystolic
murmur heard best at the cardiac apex. What is the most likely
diagnosis?

A) Viral myocarditis
B) Acute rheumatic fever
Q118 Cardiology
A 2-year-old boy is brought to the ED with fever and respiratory
distress. He recovered from a URI (rhinorrhea and congestion)
about a week ago, but fever and fatigue has persisted. Cardiac
exam shows tachycardia, an S3 gallop, and a holosystolic
murmur heard best at the cardiac apex. What is the most likely
diagnosis?

A) Viral myocarditis
B) Acute rheumatic fever
Q118 Cardiology

A) Viral myocarditis - viral prodrome (upper URI), then chest pain


and respiratory distress (from acute HF and pulmonary edema);
dilated cardiomyopathy with MR can cause an S3 gallop and
holosystolic murmur
B) Acute rheumatic fever - pharyngitis, NOT rhinorrhea &
congestion; typically presents with fever and arthritis weeks after
GAS infection
Q119 Cardiology
When is the murmur of mitral regurgitation louder?

A)inspiration
B)expiration
Q119 Cardiology
When is the murmur of mitral regurgitation louder?

A)inspiration
B)expiration
Q119 Cardiology
A)inspiration - left sided murmurs (e.g. TR) increase
with inspiration due to increase in venous return
B)expiration - right sided murmurs (e.g. MR, aortic
murmurs) increase with expiration, since the lungs
constrict and send blood to the left side of the heart
~helps distinguish MR from TR!~
Q120 Cardiology
A 30-year-old man complaining of headaches has
blood pressure read of 181/105. Auscultation reveals a
continuous murmur. What will likely be seen on CXR?

A) Vascular calcifications
B) Inferior costal surface erosions
Q120 Cardiology
A 30-year-old man complaining of headaches has
blood pressure read of 181/105. Auscultation reveals a
continuous murmur. What will likely be seen on CXR?

A) Vascular calcifications
B) Inferior costal surface erosions
Q120 Cardiology
A 30-year-old man complaining of headaches has
blood pressure read of 181/105. Auscultation reveals a
continuous murmur. What will likely be seen on CXR?

A) Vascular calcifications
B) Inferior costal surface erosions - due to pressure
induced enlargement of the rib arteries
Q120 Cardiology
Q121 Cardiology
A 51-year-old woman woke up an hour ago with severe, burning,
retrosternal chest pain, left arm numbness, along with
diaphoresis. Her heart sounds are normal, troponin levels are
normal, while EKG shows a normal sinus rhythm with some T
wave inversions. What is the next step?

A) Exercise EKG
B) Heparin
C) IV alteplase
Q121 Cardiology
A 51-year-old woman woke up an hour ago with severe, burning,
retrosternal chest pain, left arm numbness, along with
diaphoresis. Her heart sounds are normal, troponin levels are
normal, while EKG shows a normal sinus rhythm with some T
wave inversions. What is the next step?

A) Exercise EKG
B) Heparin
C) IV alteplase
Q121 Cardiology

A) Exercise EKG - for STABLE angina


B) Heparin - plus dual antiplatelet + beta blocker + nitrates (as
needed) + statins
C) IV alteplase - for STEMI, not UA or NSTEMI
Q122 Cardiology
A 57-year-old man presents with palpable rocking and
clicking of the sternum that has continued after his
recent CABG. What is the next step?

A) Reassurance
B) Surgery
Q122 Cardiology
A 57-year-old man presents with palpable rocking and
clicking of the sternum that has continued after his
recent CABG. What is the next step?

A) Reassurance
B) Surgery
Q122 Cardiology
A 57-year-old man presents with palpable rocking and
clicking of the sternum that has continued after his
recent CABG. What is the next step?

A) Reassurance
B) Surgery - sternal dehiscence is a complication of
cardiac surgery; requires surgery
Q123 Cardiology
A man falls 20 feet from a building and CXR shows abnormal
contour of the aorta and a widened mediastinum. Blood pressure
is normal. What is the diagnosis?

A) Aortic rupture
B) Pulmonary contusion
Q123 Cardiology
A man falls 20 feet from a building and CXR shows abnormal
contour of the aorta and a widened mediastinum. Blood pressure
is normal. What is the diagnosis?

A) Aortic rupture
B) Pulmonary contusion
Q123 Cardiology
A man falls 20 feet from a building and CXR shows abnormal
contour of the aorta and a widened mediastinum. Blood pressure
is normal. What is the diagnosis?

A) Aortic rupture - rapid deceleration can cause blunt thoracic


aortic injury. CXR will show widened mediastinum, abnormal
aortic contour, and/or left sided effusion due to hemothorax.
B) Pulmonary contusion
Q124 Cardiology
Which anti-hypertensive can cause bilateral ankle swelling?

A) ACE inhibitors
B) Hydrochlorothiazides
C) Metoprolol
D) Amlodipine
Q124 Cardiology
Which anti-hypertensive can cause bilateral ankle swelling?

A) ACE inhibitors
B) Hydrochlorothiazides
C) Metoprolol
D) Amlodipine
Q124 Cardiology
Which anti-hypertensive can cause bilateral ankle swelling?

A) ACE inhibitors - angioedema, hyperkalemia


B) Hydrochlorothiazides - electro. imbalances, high glucose
C) Metoprolol - bradyarrhythmias, fatigue
D) Amlodipine - peripheral edema in 25% of cases; can be
prevented/reduced by the addition of an ACEI or ARB
Q125 Cardiology
In cardiac tamponade, PCWP is:

A) increased
B) decreased
Q125 Cardiology
In cardiac tamponade, PCWP is:

A) increased
B) decreased
Q125 Cardiology
In cardiac tamponade, PCWP is:

A) increased - increased pericardial pressure


reduces CO and leads to obstructive shock (so RA
pressure and PCWP will be increased)
B) decreased
Q126 Cardiology
An elderly patient on digoxin may develop:

A) bradycardia and AV block


B) bleeding
Q126 Cardiology
An elderly patient on digoxin may develop:

A) bradycardia and AV block


B) bleeding
Q126 Cardiology
An elderly patient on digoxin may develop:

A) bradycardia and AV block (acute digoxin toxicity


most commonly presents with GI symptoms,
nausea, lethargy, and fatigue; chronic toxicity -
neurologic/visual/cardiac)
B) bleeding
Q127 Cardiology
A man with a history of anxiety presents with the following EKG.
What should be given?

A) lorazepam
B) adenosine commons.wikimedia.org/wiki/File:De-AW00011_%28CardioNetworks_ECGpedia%29.jpg
Q127 Cardiology
A man with a history of anxiety presents with the following EKG.
What should be given?

A) lorazepam
B) adenosine commons.wikimedia.org/wiki/File:De-AW00011_%28CardioNetworks_ECGpedia%29.jpg
Q127 Cardiology
A man with a history of anxiety presents with the following EKG.
What should be given?

A) lorazepam - panic attacks have normal P morphology


B) adenosine - for narrow QRS complex tachycardia; it slows the
sinus rate, increases AV nodal delay; it can also terminate
paroxysmal SVT's by interrupting the AV nodal reentry circuit.
Q128 Cardiology
What is the treatment for post-cardiac injury
syndrome?

A) aspirin
B) antibiotics
Q128 Cardiology
What is the treatment for post-cardiac injury
syndrome?

A) aspirin
B) antibiotics
Q128 Cardiology
What is the treatment for post-cardiac injury
syndrome?

A) aspirin - post cardiac injury syndrome is an immune


mediate pericarditis that can occur from weeks to
month after an MI; aspirin usually given
B) antibiotics - for bacterial pericarditis (rare)
Q130 Cardiology
Some time after a cardiac cath procedure, a patient develops
localized pain and swelling by his inguinal area. A continuous
bruit is heard with a palpable thrill over the site. What is the
diagnosis?

A) Femoral artery dissection


B) AV fistula
Q130 Cardiology
Some time after a cardiac cath procedure, a patient develops
localized pain and swelling by his inguinal area. A continuous
bruit is heard with a palpable thrill over the site. What is the
diagnosis?

A) Femoral artery dissection


B) AV fistula
Q130 Cardiology

AV fistula - can be caused as femoral vein is punctured


during needle insertion; inadequate hemostasis may allow
persistent bleeding, which tracks into the venous puncture,
creating an AV fistula; patients may be asymptomatic at first;
diagnosed with ultrasound; large ones may require surgical
repair
Q131 Cardiology
Which of the following complications of aortic dissection can
lead to heart failure and death:

A) acute aortic regurgitation


B) acute mitral valve regurgitation
Q131 Cardiology
Which of the following complications of aortic dissection can
lead to heart failure and death:

A) acute aortic regurgitation


B) acute mitral valve regurgitation
Q131 Cardiology
Which of the following complications of aortic dissection can
lead to heart failure and death:

A) acute aortic regurgitation - extension of a type A aortic


dissection into aortic valve annulus can cause acute AR and
heart failure
B) acute mitral valve regurgitation - not due to aortic
dissection, but rather to endocarditis related leaflet
destruction or infarction related papillary rupture
Q132 Cardiology
A woman is found with pulseless electrical activity on hospital
day 6 after enduring a lateral wall STEMI. After initiation of
ACLS protocol, what is the next best step?

A) Reassurance
B) Pericardiocentesis
Q132 Cardiology
A woman is found with pulseless electrical activity on hospital
day 6 after enduring a lateral wall STEMI. After initiation of
ACLS protocol, what is the next best step?

A) Reassurance
B) Pericardiocentesis
Q132 Cardiology
A woman is found with pulseless electrical activity on hospital
day 6 after enduring a lateral wall STEMI. After initiation of
ACLS protocol, what is the next best step?

A) Reassurance
B) Pericardiocentesis - a patient with pulseless electrical
activity a week after an MI likely suffered a LV free wall
rupture with acute cardiac tamponade; next best step is
pericardiocentesis
Q133 Cardiology
When is central venous pressure
elevated?

A) hypovolemic shock
B) cardiogenic shock
Q133 Cardiology
When is central venous pressure
elevated?

A) hypovolemic shock
B) cardiogenic shock
Q133 Cardiology
When is central venous pressure
elevated?

A) hypovolemic shock
B) cardiogenic shock - e.g., acute heart
dysfunction after blunt cardiac injury
Q134 Cardiology
A patient with untreated hyperthyroidism has new
onset fever and tachycardia, with BP = 180/105.
What cardiac hemodynamics will be seen?

A) increased CO and decreased SVR


B) decreased CO and increased SVR
Q134 Cardiology
A patient with untreated hyperthyroidism has new
onset fever and tachycardia, with BP = 180/105.
What cardiac hemodynamics will be seen?

A) increased CO and decreased SVR


B) decreased CO and increased SVR
Q134 Cardiology
Thyroid storm (e.g., new onset fever, severe
hypertension, tachycardia following surgery)
leads to reduced SVR (direct vasodilation) and a
reflexive increase in cardiac contractility
(increased CO).
Q135 Cardiology
How does squatting improve cyanosis in tetralogy
of fallot?

A) increases right to left shunting


B) decreases right to left shunting
Q135 Cardiology
How does squatting improve cyanosis in tetralogy
of fallot?

A) increases right to left shunting


B) decreases right to left shunting
Q135 Cardiology
How does squatting improve cyanosis in tetralogy
of fallot?

A) increases right to left shunting


B) decreases right to left shunting - squatting
increases afterload, which increases blood flow
across the RVOT (improves cyanosis)
Q135 Cardiology
overiding aorta

CC BY-SA 4.0 <https://creativecommons.org/licenses/by-sa/4.0>, via Wikimedia Commons


Q136 Cardiology
A patient develops the following cutaneous
findings after a cardiac cath procedure. What is
the diagnosis?

A) cholesterol embolism
B) vasculitis
Q136 Cardiology
A patient develops the following cutaneous
findings after a cardiac cath procedure. What is
the diagnosis?

A) cholesterol embolism
B) vasculitis
Q136 Cardiology
cholesterol embolism occurs when an
atherosclerotic plaque is dislodged and flows
into circulation; leads to partial/total occlusion of
arterioles, leading to tissue or organ ischemia;
supportive treatment & statin therapy for
recurrencerence
Q137 Cardiology
What is the cause of this EKG?

A) accessory AV pathway
B) cocaine

James Heilman, MD, CC BY-SA 3.0 <https://creativecommons.org/licenses/by-sa/3.0>, via Wikimedia Commons


Q137 Cardiology
What is the cause of this EKG?

A) accessory AV pathway
B) cocaine

James Heilman, MD, CC BY-SA 3.0 <https://creativecommons.org/licenses/by-sa/3.0>, via Wikimedia Commons


Q137 Cardiology
What is the cause of this EKG?

A) accessory AV pathway
WPW syndrome - EKG findings: short PR interval (<120 msec),
slurred upstroke of the QRS complex (delta wave), and widened
QRS complex. It is due to an accessory AV pathway. Patients are
susceptible to developing paroxysmal tachyarrhthmias.
B) cocaine
Q138 Cardiology
A cyanotic newborn's EKG shows tall P waves
and left axis deviation; there are decreased
pulmonary markings on CXR. What is the
diagnosis?

A) Tricuspid valve atresia


B) Total anomalous pulmonary venous return
Q138 Cardiology
A cyanotic newborn's EKG shows tall P waves
and left axis deviation; there are decreased
pulmonary markings on CXR. What is the
diagnosis?

A) Tricuspid valve atresia


B) Total anomalous pulmonary venous return
Q138 Cardiology
Tricuspid valve atresia refers
to NO tricuspid valve. Atrial
septal defect necessary for
survival (provides path for
blood returning to RA to
reach the lungs); VSD (to
allow) pulmonary flow
explains holosystolic murmur.
Q139 Cardiology
Amiodarone causes:

A) Hypothyroidism
B) Hyperthyroidism
C) either A or B AMI
Q139 Cardiology
Amiodarone causes:

A) Hypothyroidism
B) Hyperthyroidism
C) either A or B AMI
Q140 Cardiology
Tricuspid regurgitation murmur undergoes what
change upon inspiration?

A) increases
B) decreases
Q140 Cardiology
Tricuspid regurgitation murmur undergoes what
change upon inspiration?

A) increases
B) decreases
Q141 Cardiology
How is blunt thoracic aortic injury diagnosed?

A) Transthoracic echo
B) CT angiography
Q141 Cardiology
How is blunt thoracic aortic injury diagnosed?

A) Transthoracic echo
B) CT angiography
Q141 Cardiology
How is blunt thoracic aortic injury diagnosed?

A) Transthoracic echo
B) CT angiography

e.g., pseudocoarctation - upper extremity hypertension and


lower extremity hypotension; hoarse voice
Q142 Cardiology
A boxer has a brief loss of consciousness after standing
up. Upon evaluation, he has dry mucous membranes,
an elevated BUN/creatinine ratio, and hypokalemia.
What is the source of his syncopal episode?

A) HCM
B) Hypotension
Q142 Cardiology
A boxer has a brief loss of consciousness after standing
up. Upon evaluation, he has dry mucous membranes,
an elevated BUN/creatinine ratio, and hypokalemia.
What is the source of his syncopal episode?

A) HCM
B) Hypotension
Q142 Cardiology
B) Hypotension
-most likely orthostatic hypotension (in a boxer, most
likely due to hypovolemia due to fluid restriction and
diuretic use - explains the dry mucous membranes, and
elevated BUN; hypokalemia due to diuretic potassium
wasting effects)
Q143 Cardiology
BNP levels correlate with the severity of:

A) LA systolic dysfunction
B) LV systolic dysfunction
Q143 Cardiology
BNP levels correlate with the severity of:

A) LA systolic dysfunction
B) LV systolic dysfunction
Q143 Cardiology
LV systolic dysfunction - BNP is a hormone
released from ventricular myocytes in response
to high ventricular filling pressures and wall
stress (generally high in patients with CHF);
higher levels correlate with LV systolic
dysfunction
(unknown why it's low in obesity)
Q144 Cardiology
Amiodarone _ serum levels of digoxin:

A) increases
B) decreases
Q144 Cardiology
Amiodarone _ serum levels of digoxin:

A) increases
B) decreases
Q145 Cardiology
The acute management of STEMI involves:

A) cardiac catheterization
B) emergency cardiothoracic surgery
Q145 Cardiology
The acute management of STEMI involves:

A) cardiac catheterization
B) emergency cardiothoracic surgery
Q146 Cardiology
A 35-year-old woman with BP= 145/80, progressive
shortness of breath, peripheral edema, basilar lung
crackles, along with recent weight loss and
palpitations is consistent with _.

A) high output heart failure


B) pericardial effusion
Q146 Cardiology
A 35-year-old woman with BP= 145/80, progressive
shortness of breath, peripheral edema, basilar lung
crackles, along with recent weight loss and
palpitations is consistent with _.

A) high output heart failure


B) pericardial effusion
Q146 Cardiology

Hyperthyroidism is a common cause of high output


cardiac failure (e.g., decreased SVR, leads to an
increase in CO and venous return; wide pulse
pressure)
Q147 Cardiology
Surgical indication for infective endocarditis includes:

A) acute heart failure


B) abscess
C) an organism that is difficult to eradicate
D) large vegetation
E) all of the above
Q147 Cardiology
Surgical indication for infective endocarditis includes:

A) acute heart failure


B) abscess
C) an organism that is difficult to eradicate
D) large vegetation
E) all of the above
Q148 Cardiology
A patient has a persistent tachyarrhythmia causing
hemodynamic instability. What is the next step?

A) Amiodarone
B) Defibrillation
C) Cardioversion
Q148 Cardiology
A patient has a persistent tachyarrhythmia causing
hemodynamic instability. What is the next step?

A) Amiodarone
B) Defibrillation
C) Cardioversion
Q148 Cardiology
Cardioversion - delivers an electric shock synchronized
to the QRS complex (for stable patients, adenosine in
narrow complex tachy; amiodarone for monomorphic
ventricular tachy)
Q149 Cardiology
A patient with chest pain associated with an NSTEMI
would benefit from:

A) ibuprofen and colchicine


B) a P2Y12 inhibitor
Q149 Cardiology
A patient with chest pain associated with an NSTEMI
would benefit from:

A) ibuprofen and colchicine


B) a P2Y12 inhibitor
Q149 Cardiology
A patient with chest pain associated with an NSTEMI
would benefit from:

A) ibuprofen and colchicine - for pericarditis


B) a P2Y12 inhibitor - i.e., dual antiplatelet therapy
(aspirin and a P2Y12 - lowers morbidity and mortality
rates in NSTEMI)
Q150 Cardiology
In a patient with an AV fistula (in preparation for
hemodialysis), cardiac preload will be:

A) increased
B) decreased
Q150 Cardiology
In a patient with an AV fistula (in preparation for
hemodialysis), cardiac preload will be:

A) increased
B) decreased
Q150 Cardiology
In a patient with an AV fistula (in preparation for
hemodialysis), cardiac preload will be:

A) increased - can even lead to high output HF


B) decreased
Q151 Cardiology
A patient with resistant hypertension and diffuse
atherosclerosis most like has:

A) renovascular hypertension
B) primary hyperaldosteronism
Q151 Cardiology
A patient with resistant hypertension and diffuse
atherosclerosis most like has:

A) renovascular hypertension
B) primary hyperaldosteronism
Q151 Cardiology
Renovascular hypertension should be suspected in a
patient with resistant hypertension and diffuse
atherosclerosis (asyymmetric kidney size and pulmonary
edema may be seen; a continuous abdomional bruit has
a high specificity for renovascular hypertension)
Q152 Cardiology
A patient with epigastric pain and nausea, along with
hypotension and bradycardia has an EKG with T wave
inversions in leads II, III, and aVF. What is the next step?

A) upright abdominal x-ray


B) right-sided precordial EKG
Q152 Cardiology
A patient with epigastric pain and nausea, along with
hypotension and bradycardia has an EKG with T wave
inversions in leads II, III, and aVF. What is the next step?

A) upright abdominal x-ray


B) right-sided precordial EKG
Q152 Cardiology
Inferior wall MI's (EKG changes in II, III, and aVF) often
present with epigastric pain and sometimes nausea;
hypotension is a feature, avoid nitrates; when inferior wall
MI suspected, get right sided precordial EKG to evaluate
for RV involvement (ST elevation in V4R is highly
accurate in confirming RV MI).
Q153 Cardiology
A patient develops hypotension and has increased JVD a
day after undergoing a CABG surgery. What is the next
step?

A) Dobutamine
B) Echo
Q153 Cardiology
A patient develops hypotension and has increased JVD a
day after undergoing a CABG surgery. What is the next
step?

A) Dobutamine
B) Echo
Q153 Cardiology
A patient develops hypotension and has increased JVD a
day after undergoing a CABG surgery. What is the next
step?

A) Dobutamine
B) Echo - patient most likely has postoperative cardiac
tamponade (urgent echo should be done for diagnosis
and guide management)
Q154 Cardiology
An anti-arrhythmic is prescribed for paroxysmal atrial
fibrillation. As the patient undergoes a treadmill exercise
test, it is noted that as his heart rate goes up, the QRS
complexes widen. What drug was prescribed?

A) Flecainide
B) Digoxin
Q154 Cardiology
An anti-arrhythmic is prescribed for paroxysmal atrial
fibrillation. As the patient undergoes a treadmill exercise
test, it is noted that as his heart rate goes up, the QRS
complexes widen. What drug was prescribed?

A) Flecainide
B) Digoxin
Q154 Cardiology

Flecainide (and other class IC antiarrhythmics) - "use


dependence"; a progressive increase in sodium channel
inhibition with faster heart rates, leading to an increase in
QRS complex duration (can also be seen to a lesser
degree with class 1A's)
Q155 Cardiology
Several days after a valve replacement, a patient
develops fever, tachycardia, chest pain, and sternal
wound purulent discharge. What is the next step?

A) Broad spectrum antibiotics


B) Surgery
Q155 Cardiology
Several days after a valve replacement, a patient
develops fever, tachycardia, chest pain, and sternal
wound purulent discharge. What is the next step?

A) Broad spectrum antibiotics


B) Surgery
Q155 Cardiology
Several days after a valve replacement, a patient
develops fever, tachycardia, chest pain, and sternal
wound purulent discharge. What is the next step?

A) Broad spectrum antibiotics


B) Surgery - acute mediastinitis, a possible complication of
cardiac surgery (patients present with fever, chest pain, and
discharge; tx: drainage, surgical debridement, and prolonged
antibiotics)
Q156 Cardiology
Cardiac monitoring immediately preceding vasovagal
syncope shows:

A) bradycardia
B) tachycardia
Q156 Cardiology
Cardiac monitoring immediately preceding vasovagal
syncope shows:

A) bradycardia
B) tachycardia
Q156 Cardiology
Cardiac monitoring immediately preceding vasovagal syncope shows:

A) bradycardia - Vasovagal syncope occurs due to an alteration in


autonomic drive (most commonly, abrupt parasympathetic activation
leads to bardycardia with sinus arrest); the episode usually resolves in
less than a minute as cerebral perfusion is restored; most common in
younger patients with emotional stress (e.g., fear of spider)
B) tachycardia
Q157 Cardiology
A newborn has central cyanosis but normal blood
pressure in all extremities. A harsh crescendo-
decrescendo systolic murmur is heard over left upper
sternal border. What is the diagnosis?

A) Coarctation of the aorta


B) Tetralogy of fallot
Q157 Cardiology
A newborn has central cyanosis but normal blood
pressure in all extremities. A harsh crescendo-
decrescendo systolic murmur is heard over left upper
sternal border. What is the diagnosis?

A) Coarctation of the aorta


B) Tetralogy of fallot
Q157 Cardiology
A newborn has central cyanosis but normal blood
pressure in all extremities. A harsh crescendo-
decrescendo systolic murmur is heard over left upper
sternal border. What is the diagnosis?

A) Coarctation of the aorta


B) Tetralogy of fallot - murmur reflects pulmonary artery
stenosis
Q158 Cardiology
A man develops a moderate pleural effusion 2 days after
a CABG surgery. The patient has no respiratory
symptoms. What is the best next step?

A) Thoracentesis
B) Observation
Q158 Cardiology
A man develops a moderate pleural effusion 2 days after
a CABG surgery. The patient has no respiratory
symptoms. What is the best next step?

A) Thoracentesis
B) Observation
Q158 Cardiology
A man develops a moderate pleural effusion 2 days after
a CABG surgery. The patient has no respiratory
symptoms. What is the best next step?

A) Thoracentesis
B) Observation -
Almost half of patients undergoing CABG develop pleural effusions; most are
small and asymptomatic and result within 1-2 days of surgery
Q159 Cardiology
The murmur of HCM (due to left ventricular outflow
obstruction) increases with:

A) Squatting
B) Valsalva
Q159 Cardiology
The murmur of HCM (due to left ventricular outflow
obstruction) increases with:

A) Squatting
B) Valsalva
Q160 Cardiology
Small VSD's in newborns are typically hemodynamically
insignificant and close spontaneously by age 2. (T/F)

A) True
B) False
Q160 Cardiology
Small VSD's in newborns are typically hemodynamically
insignificant and close spontaneously by age 2. (T/F)

A) True
B) False
Q161 Cardiology
A 75-year-old obese man has dry scaly and itchy skin
over his legs. They appear brown and woody with
ulceration and swollen. How is the diagnosis of this
condition confirmed?

A) ABI
B) Venous doppler ultrasound
By James Heilman, MD - Own work, CC BY-SA 4.0, https://commons.wikimedia.org/w/index.php?curid=73507981
Q161 Cardiology
A 75-year-old obese man has dry scaly and itchy skin
over his legs. They appear brown and woody with
ulceration and swollen. How is the diagnosis of this
condition confirmed?

A) ABI
B) Venous doppler ultrasound
By James Heilman, MD - Own work, CC BY-SA 4.0, https://commons.wikimedia.org/w/index.php?curid=73507981
Q161 Cardiology
A 75-year-old obese man has dry scaly and itchy skin
over his legs. They appear brown and woody with
ulceration and swollen. How is the diagnosis of this
condition confirmed?
B) Venous doppler ultrasound - chronic stasis dermatitis
(scaling, superficial erosions, pitting edema; fibrosis and
woody induration in chronic condition) due to venous
insufficiency; diagnosis made clinically, confirmation with
ultrasound
Q162 Cardiology
What is the pathophysiology of fibromuscular
dysplasia?

A) atherosclerotic plaque formation


B) inflammation of arteries
C) abnormal cell development in the arterial wall
Q162 Cardiology
What is the pathophysiology of fibromuscular
dysplasia?

A) atherosclerotic plaque formation


B) inflammation of arteries
C) abnormal cell development in the arterial wall
Q162 Cardiology
Fibromuscular dysplasia

an idiopathic, non-inflammatory condition that mainly affects young


woman; involves abnormal cell development in the arterial wall;
primarily involves renal arteries (hypertension); cerebral arteries
(brain ischemia, amaurosis fugax, stroke)
Q163 Cardiology
A biphasic stridor in a baby that improves with neck
extension represents:

A) aspiration of a foreign body


B) airway compression by a vascular ring
Q163 Cardiology
A biphasic stridor in a baby that improves with neck
extension represents:

A) aspiration of a foreign body


B) airway compression by a vascular ring
Q163 Cardiology
A biphasic stridor in a baby that improves with neck
extension represents:

A) aspiration of a foreign body - in the upper airway


would cause stridor but the baby would be dying; in
lower airway wouldn't cause stridor
B) airway compression by a vascular ring
Q164 Cardiology
A pregnant woman from a developing country with a
history of recurrent sore throat now develops (at 30
weeks gestation) sudden onset cough, progressive
dyspnea, and orthopnea. What is the diagnosis?

A) mitral stenosis
B) peripartum cardiomyopathy
Q164 Cardiology
A pregnant woman from a developing country with a
history of recurrent sore throat now develops (at 30
weeks gestation) sudden onset cough, progressive
dyspnea, and orthopnea. What is the diagnosis?

A) mitral stenosis
B) peripartum cardiomyopathy
Q164 Cardiology
A pregnant woman from a developing country with a
history of recurrent sore throat now develops (at 30
weeks gestation) sudden onset cough, progressive
dyspnea, and orthopnea. What is the diagnosis?

A) mitral stenosis - Rheumatic, with pulm. edema


B) peripartum cardiomyopathy
Q165 Cardiology
5 days after an anterior wall MI a patient develops
sudden onset chest pain - he becomes agitated, and
soon becomes unresponsive with no pulse palpable.
What is the diagnosis?

A) free wall rupture


B) IV septum rupture
Q165 Cardiology
5 days after an anterior wall MI a patient develops
sudden onset chest pain - he becomes agitated, and
soon becomes unresponsive with no pulse palpable.
What is the diagnosis?

A) free wall rupture


B) IV septum rupture
Q166 Cardiology
What is the most effective way to minimize AAA
progression?

A) reduce systolic hypertension


B) quit smoking
Q166 Cardiology
What is the most effective way to minimize AAA
progression?

A) reduce systolic hypertension


B) quit smoking
Q167 Cardiology
A 45-year-old woman has an episode of syncope.
EKG shows sinus rhythm, prolonged PR interval,
prolonged QRS complex, occasional PVC's, and an
EF = 60%. What is the cause of her syncope?

A) bradyarrhythmia
B) PVC's
Q167 Cardiology
A 45-year-old woman has an episode of syncope.
EKG shows sinus rhythm, prolonged PR interval,
prolonged QRS complex, occasional PVC's, and an
EF = 60%. What is the cause of her syncope?

A) bradyarrhythmia
B) PVC's
Q167 Cardiology
A 45-year-old woman has an episode of syncope.
EKG shows sinus rhythm, prolonged PR interval,
prolonged QRS complex, occasional PVC's, and an
EF = 60%. What is the cause of her syncope?

A) bradyarrhythmia - suggested by prolonged PR


interval and IV conduction delay
B) PVC's - usually do not cause syncope
Q167 Cardiology

*All patients with unexplained syncope should get


an EKG; findings which suggest as arrhythmia as
the cause include bradycardia, sinus pauses, AV
block and prolonged QT*
Q168 Cardiology
Besides pain medication, what medication is given
as initial therapy for aortic dissection?

A) IV esmolol
B) thrombolytic therapy
Q168 Cardiology
Besides pain medication, what medication is given
as initial therapy for aortic dissection?

A) IV esmolol
B) thrombolytic therapy
Q168 Cardiology
Besides pain medication, what medication is given
as initial therapy for aortic dissection?

IV beta blockers (e.g., labetolol, esmolol) are given


in the initial management of aortic dissection (along
with pain management); ascending aortic dissection
is a surgical emergency
Q169 Cardiology
A patient presents with palpitations and her EKG
looks as follows:

What is the most likely pathophysiology?


A) disorganized atrial activity
B) 2 distinct conduction pathways in the AV node
Q169 Cardiology
A patient presents with palpitations and her EKG
looks as follows:

What is the most likely pathophysiology?


A) disorganized atrial activity
B) 2 distinct conduction pathways in the AV node
Q169 Cardiology
The most common type of paroxysmal
supraventricular tachycardia is AV nodal reentrant
tachycardia, in which there are narrow QRS
complexes with a regular rhythm and indistinct P
waves; can be terminated with adenosine; catheter
ablation may be considered as long term therapy
Q170 Cardiology
A patient with an inferior wall MI now as
hypotension. What is the cause?

A) increased intrapericardial pressure


B) decreased RV contractility
Q170 Cardiology
A patient with an inferior wall MI now as
hypotension. What is the cause?

A) increased intrapericardial pressure


B) decreased RV contractility
Q171 Cardiology
What is the treament for monomorphic ventricular
tachycardia in a stable patient?

A) cardioversion
B) carotid massage
C) amiodarone
Q171 Cardiology
What is the treament for monomorphic ventricular
tachycardia in a stable patient?

A) cardioversion
B) carotid massage
C) amiodarone
Q172 Cardiology
A patient underwent femoral embolectomy and now has
swelling of the right calf, with paresthesia along with severe
pain that is worse with passive range of motion. There are
also sensory and motor deficits, but dorsalis pedis and
posterior tibial pulses are palpable. What is the diagnosis?

A) compartment syndrome
B) venous thrombosis
Q172 Cardiology
A patient underwent femoral embolectomy and now has
swelling of the right calf, with paresthesia along with severe
pain that is worse with passive range of motion. There are
also sensory and motor deficits, but dorsalis pedis and
posterior tibial pulses are palpable. What is the diagnosis?

A) compartment syndrome
B) venous thrombosis
Q172 Cardiology

Compartment syndrome can occur after reperfusion of an


ischemic limb (e.g., embolectomy) - features include sensory
and motor deficits, along with pain upon passive range of
motion; tx: emergency fasciotomy
Q173 Cardiology
In septic shock, mixed venous oxygen
saturation is:

A) increased
B) decreased
Q173 Cardiology
In septic shock, mixed venous oxygen
saturation is:

A) increased
B) decreased
Q173 Cardiology
Septic shock leads to decreased SVR due to peripheral
vasodilation; initially CO is increased in an attempt to
maintain peripheral tissue perfusion, leading to elevated
mixed venous oxygen saturation, since the tissues can't
extract oxygen quickly enough as blood flows through the
capillaries; in late septic shock, CO decreases as
myocardial contractile dysfunction develops
Q174 Cardiology
A patient has a procedure in which a pacemaker is
implanted and now has signs of heart failure. A
holosystolic murmur is best heard at the left sternal
border. What is the diagnosis?

A) Tricuspid regurgitation
B) Ventricular septal defect
Q174 Cardiology
A patient has a procedure in which a pacemaker is
implanted and now has signs of heart failure. A
holosystolic murmur is best heard at the left sternal
border. What is the diagnosis?

A) Tricuspid regurgitation
B) Ventricular septal defect
Q175 Cardiology
A 67-year-old lady has an episode of syncope. This is
what her EKG looks like.

CC BY-SA 3.0, https://commons.wikimedia.org/w/index.php?curid=22055720

What is the next step?


A) atropine
B) norepinephrine
Q175 Cardiology
A 67-year-old lady has an episode of syncope. This is
what her EKG looks like.

CC BY-SA 3.0, https://commons.wikimedia.org/w/index.php?curid=22055720

What is the next step?


A) atropine
B) norepinephrine
Q176 Cardiology
A 20-hour-old girl is evaluated for worsening cyanosis and rapid
breathing. Pulse ox = 79%, which does not improve with pure
oxygen. She is hypotensive and tachycardic, but no murmurs
are present and breath sounds are normal. Echo is pending.
What is the next best step?

A) surfactant
B) prostaglandin E1
Q176 Cardiology
A 20-hour-old girl is evaluated for worsening cyanosis and rapid
breathing. Pulse ox = 79%, which does not improve with pure
oxygen. She is hypotensive and tachycardic, but no murmurs
are present and breath sounds are normal. Echo is pending.
What is the next best step?

A) surfactant
B) prostaglandin E1
Q176 Cardiology
A 20-hour-old girl is evaluated for worsening cyanosis and rapid
breathing. Pulse ox = 79%, which does not improve with pure
oxygen. She is hypotensive and tachycardic, but no murmurs
are present and breath sounds are normal. Echo is pending.
What is the next best step?

A) surfactant
B) prostaglandin E1 - maintains patency of the PDA (e.g., in
coarctation of the aorta)
Q176 Cardiology
A 20-hour-old girl is evaluated for worsening cyanosis and rapid
breathing. Pulse ox = 79%, which does not improve with pure
oxygen. She is hypotensive and tachycardic, but no murmurs
are present and breath sounds are normal. Echo is pending.
What is the next best step?
Patients with suspected severe aortic coarctation should
A) surfactant immediately receive prostaglandin E1 to maintain patency
of the DA, by allowing blood from the pulmonary artery to
B) prostaglandin E1 supply the descending aorta (R-L shunting), to restore
distal perfusion!
Q177 Cardiology
A 64-year-old man with substernal chest pain is found to have
decompensated heart failure. BP = 108/71 An S3 is heard on
exam as well as bibasilar crackles halfway up the lung fields.
The patient is given aspirin, clopidogrel, a statin, and
anticoagulation. What else should be given?

A) Metoprolol
B) Furosemide
Q177 Cardiology
A 64-year-old man with substernal chest pain is found to have
decompensated heart failure. BP = 108/71 An S3 is heard on
exam as well as bibasilar crackles halfway up the lung fields.
The patient is given aspirin, clopidogrel, a statin, and
anticoagulation. What else should be given?

A) Metoprolol
B) Furosemide
Q177 Cardiology
A 64-year-old man with substernal chest pain is found to have
decompensated heart failure. BP = 108/71 An S3 is heard on
exam as well as bibasilar crackles halfway up the lung fields.
The patient is given aspirin, clopidogrel, a statin, and
anticoagulation. What else should be given?

A) Metoprolol - NOT in decompensated HF


B) Furosemide - for decompensated HF
Q178 Cardiology
The most common peripheral artery aneurysms are:

A) popliteal and femoral artery


B) carotid and vertebral artery
Q178 Cardiology
The most common peripheral artery aneurysms are:

A) popliteal and femoral artery


B) carotid and vertebral artery
Q179 Cardiology
A patient with a history of radiation therapy now experiences
peripheral edema, ascites, along with dyspnea on exertion. JVP
is elevated. What is the source?

A) portal vein thrombosis


B) venous valve incompetence
C) constrictive pericarditis
Q179 Cardiology
A patient with a history of radiation therapy now experiences
peripheral edema, ascites, along with dyspnea on exertion. JVP
is elevated. What is the source?

A) portal vein thrombosis


B) venous valve incompetence
C) constrictive pericarditis
Q180 Cardiology
A baby born several hours ago ago presents with cyanosis
(since birth), tachypnea, a normal S1 and a single loud S2. Pure
oxygen does not improve the cyanosis. What is the diagnosis?

A) Atrial septal defect


B) Hypoplastic left heart syndrome
C) Tetralogy of fallot
D) Transposition of the great arteries
Q180 Cardiology
A baby born several hours ago ago presents with cyanosis
(since birth), tachypnea, a normal S1 and a single loud S2. Pure
oxygen does not improve the cyanosis. What is the diagnosis?

A) Atrial septal defect


B) Hypoplastic left heart syndrome
C) Tetralogy of fallot
D) Transposition of the great arteries
Q181 Cardiology
All patients age > 40 with DM should start lipid
lowering therapy with a statin, regardless of baseline
lipid levels. T/F.

A) True
B) False
Q181 Cardiology
All patients age > 40 with DM should start lipid
lowering therapy with a statin, regardless of baseline
lipid levels. T/F.

A) True
B) False
Q181 Cardiology
All patients age > 40 with DM should start lipid
lowering therapy with a statin, regardless of baseline
lipid levels. T/F.

A) True - and if under 40 with additional


cardiovascular risk factors
B) False
Q182 Cardiology
A complication of infective endocarditis is acute mitral
regurgitation. T/F.

A) True
B) False
Q182 Cardiology
A complication of infective endocarditis is acute mitral
regurgitation. T/F.

A) True
B) False
Q183 Cardiology
A patient experiences syncope. Here is his EKG.
What is the treatment?

A) Metoprolol
B) Pericardiocentesis
Q183 Cardiology
A patient experiences syncope. Here is his EKG.
What is the treatment?

A) Metoprolol
B) Pericardiocentesis
Q184 Cardiology
How is alcoholic cardiomyopathy diagnosed?

A) angiography
B) exclusion
Q184 Cardiology
How is alcoholic cardiomyopathy diagnosed?

A) angiography
B) exclusion
Q185 Cardiology
A baby is born with a VSD. What is occuring in the
baby's heart?

A) increased preload
B) right to left shunting
Q185 Cardiology
A baby is born with a VSD. What is occuring in the
baby's heart?

A) increased LA preload
B) right to left shunting
Q186 Cardiology
A newborn of a diabetic number has respiratory
distress with a systolic murmur and a thickened IV
septum. What will most likely eventually occur?

A) Need for surgery


B) Spontaneous regression
Q186 Cardiology
A newborn of a diabetic number has respiratory
distress with a systolic murmur and a thickened IV
septum. What will most likely eventually occur?

A) Need for surgery


B) Spontaneous regression
Q187 Cardiology
Joint laxity, poor wound healing, and skin
hyperextensibility in a normal height patient is seen
in:

A) Osteogenesis imperfecta
B) Marfan syndomre
C) Ehlers-Danlos syndrome
Q187 Cardiology
Joint laxity, poor wound healing, and skin
hyperextensibility in a normal height patient is seen
in:

A) Osteogenesis imperfecta
B) Marfan syndomre
C) Ehlers-Danlos syndrome
Q188 Cardiology
What is the treatment for congenital long QT
syndrome?

A) quinidine
B) propranolol
Q188 Cardiology
What is the treatment for congenital long QT
syndrome?

A) quinidine
B) propranolol
Q189 Cardiology
Which class of medications does NOT improve
mortality in patients with HF with reduced EF?

A) ACE inhibitors
B) beta blockers
C) mineralocorticoid receptor antagonists
D) SGLT2 inhibitors
E) CCB's
Q189 Cardiology
Which class of medications does NOT improve
mortality in patients with HF with reduced EF?

A) ACE inhibitors
B) beta blockers
C) mineralocorticoid receptor antagonists
D) SGLT2 inhibitors
E) CCB's
Q190 Cardiology
A patient has an elevated BUN, along with pleuritic
chest pain and a friction rub. What is the diagnosis?

A) Pericarditis
B) Pulmonary embolism
Q190 Cardiology
A patient has an elevated BUN, along with pleuritic
chest pain and a friction rub. What is the diagnosis?

A) Pericarditis
B) Pulmonary embolism
Q191 Cardiology
Which of the following have a cardiovascular benefit
for patients with hypertriglyceridemia already taking
a statin?

A) Add a fibrate
B) Reduce alcohol intake
Q191 Cardiology
Which of the following have a cardiovascular benefit
for patients with hypertriglyceridemia already taking
a statin?

A) Add a fibrate
B) Reduce alcohol intake
Q192 Cardiology
Syncope that occurs suddenly is most likely due to
a:

A) cardiac or neurological cause


B) toxin or metabolic problem
Q192 Cardiology
Syncope that occurs suddenly is most likely due to
a:

A) cardiac or neurological cause


B) toxin or metabolic problem
Q193 Cardiology
Syncope due to ventricular tachycardia or fibrillation
in a man from Asia (with a right BBB seen on EKG)
is treated with:

A) metoprolol
B) AICD placement
Q193 Cardiology
Syncope due to ventricular tachycardia or fibrillation
in a man from Asia (with a right BBB seen on EKG)
is treated with:

A) metoprolol
B) AICD placement
Q194 Cardiology
A 55-year-old woman develops chest pain, dyspnea,
and ST elevations in V2-V4 after hearing her son
died. Troponins are elevated but coronary
angiography is normal. EKG revels LV "ballooning".
What is the mechanism?

A) catecholamine release
B) plaque rupture
Q194 Cardiology
A 55-year-old woman develops chest pain, dyspnea,
and ST elevations in V2-V4 after hearing her son
died. Troponins are elevated but coronary
angiography is normal. EKG revels LV "ballooning".
What is the mechanism?

A) catecholamine release
B) plaque rupture
Q195 Cardiology
A 74-year-old comes in for a 5 month follow up after an
uneventful cardiac bypass surgery. Physical exam is normal
except an irregular pulse. His EKG shows the following:
CC BY-SA 3.0, https://commons.wikimedia.org/w/index.php?curid=22054465

What is the best next step?


A) Begin verapamil
B) Oral anti-coagulant
Q195 Cardiology
A 74-year-old comes in for a 5 month follow up after an
uneventful cardiac bypass surgery. Physical exam is normal
except an irregular pulse. His EKG shows the following:
CC BY-SA 3.0, https://commons.wikimedia.org/w/index.php?curid=22054465

What is the best next step?


A) Begin verapamil
B) Oral anti-coagulant
Q195 Cardiology
A 74-year-old comes in for a 5 month follow up after an
uneventful cardiac bypass surgery. Physical exam is normal
except an irregular pulse. His EKG shows the following:

*Bottom line: Just like a-fib, atrial flutter carries a risk of


arterial thromboembolization and should therefore also
managed with chronic anticoagulation.*
Q196 Cardiology
Which lipid lowering medication can lead to gout?

A) Statins
B) Niacin
C) Fibric acid derivatives
D) Cholestyramine
Q196 Cardiology
Which lipid lowering medication can lead to gout?

A) Statins
B) Niacin
C) Fibric acid derivatives
D) Cholestyramine
Q196 Cardiology
Which lipid lowering medication can lead to gout?

A) Statins - AST/ALT elevation, myositis


B) Niacin - elevation of glucose/uric acid, pruritis
C) Fibric acid derivatives - risk of myositis if w/ statins
D) Cholestyramine - flatus and abdominal cramping
Q197 Cardiology
A 67-year-old woman comes to the ED with crushing chest
pain; which of the following EKG findings indicates the
worst prognosis?

A) ST elevations in leads II, III, aVF


B) ST elevations in leads V2-V4
C) ST depressions in leads V1, V2
D) PVC's
Q197 Cardiology
A 67-year-old woman comes to the ED with crushing chest
pain; which of the following EKG findings indicates the
worst prognosis?

A) ST elevations in leads II, III, aVF


B) ST elevations in leads V2-V4
C) ST depressions in leads V1, V2
D) PVC's
Q198 Cardiology
In decompensated heart failure, pulmonary
edema is always present.

A) True
B) False
Q198 Cardiology
In decompensated heart failure, pulmonary
edema is always present.

A) True
B) False
Q198 Cardiology
In decompensated heart failure, pulmonary
edema is always present.

A) True
B) False - *patients with
progressive decompensation
may have no pulmonary edema*
Q199 Cardiology
A 69-year-old woman comes to the ED with acute
onset SOB, RR = 39/min, rales, S3 gallop, and JVD.
What is the best first step?

A) echo
B) IV furosemide
C) metoprolol
Q199 Cardiology
A 69-year-old woman comes to the ED with acute
onset SOB, RR = 39/min, rales, S3 gallop, and JVD.
What is the best first step?

A) echo
B) IV furosemide
C) metoprolol
Q200 Cardiology
What is the most common cause of death from
CHF?

A) MI
B) emboli
C) pulmonary edema
D) arrhythmia
Q200 Cardiology
What is the most common cause of death from
CHF?

A) MI
B) emboli
C) pulmonary edema
D) arrhythmia/sudden death

You might also like